Download as pdf or txt
Download as pdf or txt
You are on page 1of 96

KEJAKSAAN AGUNG LAW ACADEMY

Introduction to Principles of Business


Laws (English law & New York law)
March – December 2024

PART 1: CONTRACT LAW

A.M. Alfridijanta
(New York attorney/UK solicitor)
alfrid.houston@gmail.com page 1/95
FORMATION; PERFORMANCE BREACH AND DISCHARGE;

DEFENCES, PAROLE EVIDENCE, REMEDIES, THIRD PARTY RIGHTS

I. INTRODUCTION
A. A Roadmap for the three Contracts lectures
1. Day 1: Formation – offer acceptance and consideration
2. Day 2: statute of fraud, interpretation and contract performance
3. Day 3: defenses, remedies, third party issues

II. SOURCES OF CONTRACT LAW

A. The Common Law of Contracts:


1. The common law of contracts:
Default law governing contractual relations
B. The Uniform Commercial Code
1. The UCC governs “sales of goods”
a. Sales are transactions in which a seller transfers title of goods to a buyer for value. A complete permanent
outright conveyance
(1) Sales are not leases or bailment or transactions short of outright sales

b. Goods are broadly defined as any moveable item


(1) Examples of goods in the Code include:
(a) growing crops
(b) uncut timber
(c) unborn animals
(2) Goods do not include:
(a) intangibles
(b) money
(c) the assignment of a legal claim
(d) services
(e) real property
c. In hybrid cases, which involve both the sale of goods and a services contract, the question arises as to which
source of contract law should apply.

EXAMPLE:
Bob purchases a car stereo, and the mechanic also installs it.
(1) Majority rule:
Determine which law applies by the predominant purpose of the transaction – is it more about the
good part or more about the service?

alfrid.houston@gmail.com page 2/95


EXAMPLE:
A purchase of a water heater where the merchant has it delivered and installed, is a sale of goods
and the service is incidental.
EXAMPLE:
A contract with an artist to paint a portrait is clearly a service. – mainly about the service
(a) To determine the predominant purpose of a contract, the following factors are used:
1) Factor #1:
The language of the contract
2) Factor #2:
The nature of the supplier’s business
3) Factor #3:
The relative values of the good versus the service
(2) Minority rule:
Apply the UCC to the goods portion of the transaction and the common law to the service portion

d. Besides the case of a hybrid transaction, another hard case involves:


computer software
(1) Majority Rule:
Treated as goods under the UCC
(2) The Uniform Computer Information Transactions Act displaces the UCC in Maryland and Virginia in this
area.
C. Interplay between Sources
1. Common Law Filling in the Gaps of the UCC
a. Even in a sale of goods, the common law will apply unless:
the UCC provides contrary provisions.
(1) Examples of common law filling in UCC gaps:
Defenses eg fraud duress and incapacity – the UCC is silent on them.
(2) Examples of UCC displacing the common law:
Firm offers and the battle of the forms rules.

D. Basic Definitions and Concepts of Contract Law


1. Contractual Obligations
a. There are three general categories of contractual obligations.
(1) Express contractual obligations are found where the parties make oral and written expressions of their
commitments.

(2) Implied-in-fact contractual obligations are agreements formed by conduct rather than words.

EXAMPLE:
Homeowner hires a plumber to fix a leak, but because of the urgency of the service, the parties do
not discuss the price of the work performed. Upon completion of the work, Homeowner has an
implied-in-fact obligation to pay the plumber the reasonable value of the services rendered.

alfrid.houston@gmail.com page 3/95


(3) An implied-in-law contractual obligation arises where:
One party bestows a benefit on another and it is unjust for the recipient to retain the benefit
without paying (unjust enrichment/restitution).

EXAMPLE:
Emergency Services: A surgeon who performs emergency surgery on an unconscious patient creates
an implied-in-law obligation to the patient.
EXAMPLE:
Benefits Conferred by Mistake: A merchant who mistakenly delivers goods to the wrong party may
create an implied-in-law obligation on the wrong party.

b. Quantum Meruit – implied contracts


(1) The traditional definition:
The cause of action for an implied in fact contract brought in order to recover the reasonable
value of the benefits provided.

EXAMPLE:
The plumber who was hired to fix Homeowner’s leak without first discussing the price.

(2) But, importantly, the recent usage: courts have allowed this to be used in implied in law claims as well (i.e.
when price wasn’t specifically discussed).

EXAMPLE:
The surgeon with a recalcitrant patient.
EXAMPLE:
The merchant who mistakenly delivers goods to the wrong party.

2. Signed Writing Requirements


a. A number of rules in contract law require a “signed writing” in order to create an enforceable legal obligation.
(1) Examples include:
(a) rules pertaining to a merchants’ firm offers
(b) statute of fraud requirements

b. The contemporary problem of electronic contracting:

(1) The Signing Requirement


(a) electronic signatures are legally effective in the vast majority of US jurisdictions

(2) The Writing Requirement


(a) email and the like are also sufficient to satisfy the writing requirements of contract law

alfrid.houston@gmail.com page 4/95


III. FORMATION

A. Problem That Offer and Acceptance Law Is Designed to Address


1. The “Volley”
a. In many cases, parties to a would-be contract negotiate in advance of reaching agreement, engaging in a
communications “volley” where inquiries, proposals, and counter-proposals are exchanged.
b. The key question to be answered:
At what point do parties have a legally enforceable contract?
B. The Offer
1. In order to constitute an offer, a party’s communication must meet two requirements.
a. First Requirement:
An outward manifestation – i.e. willingness
(1) The manifestation can be oral, written or made via conduct

EXAMPLE:
A newspaper seller on the corner with a stack of newspapers and hands them to people who pay the
posted price.

(2) An offer is not about:


Inward thoughts or subjective intentions which are irrelevant unless reasonably apparent to the
other party.

EXAMPLE:
A seemingly serious offer to sell real property made in secret jest is nonetheless an offer.
EXAMPLE:
A proposal to sell at a price that a reasonable person would regard as “much too good to be true”
(e.g., “new HDTVs for $8.99”) DOES NOT constitute an offer. – reasonably apparent

b. Second Requirement:
signal that acceptance will conclude the deal (i.e. they just have to say yes)

EXAMPLE:
“I will sell you my car if you’ll pay me $5,000 cash.” This is an offer because it expresses a willingness to
conclude the deal if the other party pays the required $5,000.
EXAMPLE:
“Yes, I’d be willing to sell you my car, but what are you willing to pay for it?” This is NOT AN OFFER because the
communicating party is obviously reserving the right to decide whether she likes the price suggested by the other
party.
(1) Communications that withhold the privilege of further assent fall short of constituting an offer.
(2) There are a couple of categories of communications that are not offers, that withhold the privilege of further
assent, and that will help you sort through these “volley” problems.

(a) Preliminary Negotiations

alfrid.houston@gmail.com page 5/95


1) generic term that applies to the give and take that occurs during bargaining

(b) Invitation for an Offer


1) advanced stage of preliminary negotiations where the communiting party is closing on a deal but
wants the other party to commit first.

EXAMPLE:
“So, how much are you willing to pay me for these goods?”

2. The Problem of Multiple Offerees – comes up regularly on MBE


a. When a question involves a party’s communication proposing a deal to two or more persons at the same time, a
multiple offerees issue arises.
b. There are three situations where this can occur:
(1) First situation:
Commercial ads – recurring on the MBE
(a) The American advertising rule:
Ads catalogues and price lists are not offers but are instead invitations for offers

(b) The reason for the rule is:


Responses may exceed the available supply of goods and services.

EXCEPTION:
Language that identifies who gets the limited supply of goods even if there is an excess of demand.-
that’s an offer.

EXAMPLE:
Advertisements that say, “first come, first served” or “first 10 customers only” are construed as offers.

(2) Second Situation:


Reward offers – recurring on the MBE
(a) Generally, reward offers are treated as:
Offers, as are communications that promise a bounty in exchange for the performance of a specified
task.

(b) There are two types of reward offers.


1) Self-limiting rewards:
Offers that indicate that the task to be rewarded can be performed only once.

EXAMPLE:
“Will pay $100 for finding my lost dog.”

2) Open-field rewards:
Indicate the task to be performed can be performed by multiple parties.

EXAMPLE:
The Carbolic Smoke Ball case, where the purveyor of a medical preparation promised to pay $100
to anyone who used the preparation as directed and nonetheless came down with the flu.

alfrid.houston@gmail.com page 6/95


EXCEPTION:
Language in the offer that specified a limitation on how many can accept the offer.

EXAMPLE:
“Free Joe Six-Pack Action Figure! Send us proof of purchase labels from six cases of your favorite
Anheuser-Busch product together with a self-addressed stamped envelope. Offer good while
supplies last!” – here, limitation.

(3) Third Situation: auctions - comes up every once in a while


(a) The general rule:
Auctioneer is inviting offers; responsive bids are the offers.

EXCEPTION:
If the auction is held “without reserve,” then:
The auctioneer is making an offer to sell to the highest bidder.

3. The Legal Effect of an Offer


a. An offer creates the power of acceptance in the offeree.
b. Generally, there are four ways to terminate the power of acceptance:
lapse, death or incapacity, revocation by offeror or rejection by offeree. (if they happen before the power of
acceptance exercised – too late, no contract).

(1) Termination by Lapse of Time


(a) An offer lapses after:
The time stated in the offer, or after a reasonable time, if no time stated.

(b) The reasonable time determination is based on the following factors:


1) Factor #1:
Subject matter and market conditions (something stable which can last for eg)
2) Factor #2:
Degree of urgency and means of transmission
(c) Face-to-Face Conversation Rule:
An offer made in a face to face conversation generally lapses at the end of the conversation unless
stated otherwise.

(2) Termination by Death or Incapacity of Either Party


(a) Death:
The offeror or offeree dies. Dead man can’t make contracts

(b) Incapacity:
If either party becomes mentally incompetent, the power of acceptance is terminated.

alfrid.houston@gmail.com page 7/95


(3) Termination by Offeror’s Revocation – offer withdrawn
(a) The American rule of the free revocability of offers:
An offeror may revoke an offer at any time and for any reason, so long as: req 1 and req 2 met

1) Requirement #1:
Timing – must be revoked before offer is accepted

2) Requirement #2:
Must be communicated to the offeree.

a) Direct revocation:
oferror directly communicates to the offeree an intent to withdraw an offer.

b) Indirect revocation:
i) Requirement #1:
the offeror takes some action that is inconsistent with the intention to go through the offer.

EXAMPLE:
While deciding whether to accept an offer to sell you a car, you learn from a friend that the
car was sold to someone else.
ii) Requirement #2:
the offeree learns about such action from a reliable source.

HYPOTHETICAL

A offers to sell real property to B. While B is considering the offer, A sells the property to C. Oblivious to
the third-party sale, B sees A on the street and yells, “I accept your offer!” Is A now contractually bound
to sell the property to B?

Yes, as B hasn’t learnt about it.

(b) Revocation of an Offer Made to Multiple Offerees


1) Functional Equivalents Rule:
The oferor revokes by communicating the revocation by a functionally equivalent manner as the
offer was made.

EXCEPTION:
If better means to communicate the revocation that is reasonably available, then those means must
be used to revoke.
alfrid.houston@gmail.com page 8/95
2) Legal effect of functionally equivalent revocation:
Terminates the power of acceptance even if the offeree is unaware of the revocation.

(c) Preventing Revocation: Option Contracts and Firm Offers – very important!
1) American Rule of Free Revocability of Offers
a) The rule of Dickinson v. Dodds:
at common law an offeror can revoke even if he had expressly promised the oferee that he
would hold the offer open.

2) Under contemporary law, there are two ways to prevent revocation of an offer: (i) common law
option contract or (ii) firm offer under the UCC.
3) Preventing Revocation by Means of a Common Law Option Contract
a) First element:
an offer
b) Second element:
a subsidiary promise to keep that offer open
i) “Sell by” date:
such a date may not be a subsidiary promise to keep the offer promise, but may be a
lapse date!!!

c) Third element:
some valid mechanism for enforcing the subsidiary promise

i) Consideration is the most common way to enforce the subsidiary promise.

EXAMPLE:
“I hereby offer to sell you Blackacre for $10,000, and in consideration for the $100 received, I
hereby grant you a 30-day option on the deal.”

4) Nuances of Option Contracts


a) Signed Writing with False Recital – i.e. it says money was paid for the subsidiary
promise but in fact no money paid.
i) Majority rule is:
the recital is rebuttable.
ii) Minority rule is:
option contracts will be enforced even if the recital for consideration for the option is false.

b) Promissory estoppel:
courts will sometimes enforce a subsidiary promise to keep an offer open if there has been
detrimental reliance.

5) Special Rule for Construction Contracts


a) A problem occurs when a general contractor formulates a bid to an owner and relies on
subcontractors’ bids before he wants to accept them. The general contractor will have problem
if subcontractors revoke before owner awards bid.
alfrid.houston@gmail.com page 9/95
b) The majority rule is where a general contractor uses a particular subcontractor’s bid to
formulate his own:
promissory estoppel applies to protect the general contractor from revocations from the
subcontractor.

6) Preventing Revocation by Means of a “Firm” Offer under the UCC


a) Under the UCC Section 2-205, a merchant can make a firm offer (an irrevocable offer) to either
buy or sell goods without consideration so long as three conditions are met:
i) Condition #1:
the offer to buy or sell is made by a merchant (someone in business).

ii) Condition #2:


offer is made in a writing signed by the merchant

iii) Condition #3:


offer expressly states by its terms that it will be held open.

b) A firm offer that meets all of these requirements becomes irrevocable for:
either the period of time stated, or if no time is specified, then for reasonable time.

c) Under the UCC, the shelf life of a firm offer can be:
no more than 3 months.
d) Three-Month Rule:
if a firm offer is more than 3 months, it is only irrevocable for the first 3 months.

e) Offers for longer than three months:


need consideration to be binding as an option contract.

RECAP QUESTIONS:
Please take a few minutes and answer the following questions. Try to answer these questions without looking at your Bar
Notes. If you must go back to your notes, try rewriting the answer in your own words. This exercise will help improve your
memory retention of these concepts.

1. What are the three factors used to determine whether the predominant purpose of a contract is for the sale of goods or not?

The language of the contract; the nature of the supplier’s business; the price for the goods vs the price for the service.

2. What are the two requirements of a valid offer?

An outward manifestation and a signal that acceptance will conclude the deal.

3. Describe the rule regarding commercial advertisements as offers.


Commercial ads are not offers, they are invitation for an offer. Unless language …

4. Describe the four ways to terminate the power of acceptance of an offer.


Death or incapacity, lapse of time, revocation by the offeror, rejection by the offeree.

alfrid.houston@gmail.com page 10/95


(4) Termination by Offeree’s Rejection
(a) The power of acceptance can also be terminated if the offeree refuses to accept the offer. There are
three ways that rejection can be effected:
1) First way:
Outright rejection
2) Second way:
Rejection via counteroffer
a) A counteroffer constitutes: - tested on the MBE!
a rejection while also creates a new offer

EXAMPLE:
“I am not willing to pay $10,000 for the car, but I would happily buy your car for $9,000.”
b) An exception or nuance—the Mere Inquiry Rule: Polite suggestion
an offeree may test the waters by making a mere enquiry about the offeror’s willingness to
negotiate without creating a counter offer and terminating the power of acceptance.

EXAMPLE:
“$10,000 isn’t out of the question, but it’s a little high, given the age of the car. Would you be
willing to consider a lower offer?”
3) Third way:
Rejection via non conforming acceptance.
a) The Mirror Image Rule
i) Corollary to the notion that “the maker is the master of the offer.”
ii) The common law “mirror image” rule requires that:
acceptance must mirror the terms of the offer and any variation results in a counter offer
and thus rejection of the initial offer (e.g. if you add things to your “acceptance”)

EXAMPLE:
X offers to buy goods from Y, Y says OK, and says he expects payment in 30 days. This
will be a violation of the mirror image rule, and thus a counteroffer.
c. Revival of the Offer after Rejection or Lapse – sometimes tested
(1) an offeror can revive a terminated offer by language or conduct restating the offer or giving more time to
decide.

4. Offer and Acceptance under Unilateral Contracts


a. Distinguish between bilateral and unilateral contracts.

Bilateral Contracts Unilateral Contracts

I make a promise and seek from you: a I make a promise and seek from you: only
promise in exchange. performance in exchange.
Once promises are exchanged: parties are The offeror is bound only when: offeree’s
bound. performance is completed and the offeree
is never bound.
Bilateral – two promises Unilateral – one promise

alfrid.houston@gmail.com page 11/95


b. Unless the terms of the offer state otherwise, the offeree can accept either by promising or performing. So, when
looking for a unilateral contract, one ought to carefully consider both the language and circumstances.
HYPOTHETICAL

A says to his old friend B, “I’ll sell you my car for $10,000, but due to your credit problem, you are going
to have to pay me the money or I will sell it to someone else.” Will a promissory acceptance from B form
a binding contract with A? No, only performance will suffice – a unilateral contract.

c. Revocation of the Offer in a Unilateral Contract – very highly tested!!


(1) Under common law:
An offeror is free to revoke the unilateral offer up until the offeree actually completed performance.

(2) Under the modern rule: - VERY HIGHLY TESTED – APPLY IN EXAM
Once the offeree begins performance, an option contract is created and the oferor may not
revoke.

(a) Note: While courts are split, apply the modern rule unless told otherwise.
(b) With the modern rule, there are two things that are unchanged from the common law
1) The offeree of a unilateral contract is still:
Free to abandon the performance at any time or not undertake performance at all
2) Acceptance of the offer is still:
Effective upon the completion of performance in accordance with the terms of the offer.

(3) An important nuance: - often tested


(a) Beginning of performance vs. mere preparations:
Mere preparations do not create an option contract; only the beginning of actual performance.

EXAMPLE:
X offers Y $100 to “ride my horse Bronco for a minute,” and while Y is putting on his boots X revokes.
C. Acceptance under Common Law
1. There are only two general requirements to constitute effective acceptance.
a. First requirement:
the acceptance must mirror the terms of the offer
b. Second requirement:

alfrid.houston@gmail.com page 12/95


the acceptance must be communicated to the offeror.
(1) If the offer stipulates a particular means of communicating acceptance:
That becomes the required means of acceptance.

(2) If the offer is silent as to the means of communication:


The offeree is free to use any reasonable means of transmission.
(a) Unless the circumstances indicate otherwise, a means of transmission is reasonable if it is: (one of three
applies?)
1) the means used by the offeror
2) means customarily used in similar transactions
3) means equivalent in speed and reliability to the means used by the offeror
(3) There are three exceptions to the requirement that acceptance be communicated.
(a) The first exception to the requirement that acceptance be communicated:
Acceptance by silence
1) Default rule:
Offeree’s silence in response to an offer doesn’t constitute acceptance.
2) The “acceptance by silence” exception is in play in the following situations:
a) Situation #1:
where the offeree takes the benefit of the offeror’s services with a reasonable opportunity to
reject them and a reason to know compensation was expected.

EXAMPLE:
A father hires a golf pro to provide one of his sons a lesson. Father brings both of his sons to
the driving range. The golf pro provides the second son a lesson as well.
b) Situation #2:
offeror has given the offeree a reason to understand that acceptance may be communicated
by silence

EXAMPLE:
An insurance company notifies A that her policy will be automatically renewed unless it hears
otherwise from her. A remains silent, intending to accept, and accordingly her acceptance is
effected by her silence.
EXAMPLE:
Same facts as above, except that A does not intend to accept. In these circumstances, her
silence will not operate as acceptance.
c) Situation #3:
previous dealings or other circumstances it is reasonable that the offeree should notify the
offeror if he doesn’t intent to accept.

EXAMPLE:
A newspaper sends A a written notice that it will continue delivering the paper after the
subscription has expired unless it hears otherwise from A. When the subscription has
previously been renewed in this manner without incident, A’s silence in the face of notice
constitutes acceptance.
(b) The second exception to the requirement that acceptance be communicated:

1) In a unilateral contract setting:

alfrid.houston@gmail.com page 13/95


Acceptance is affected only by completing performance , communication is not required for
formation, unless the offer provides otherwise.

HYPOTHETICAL

Professor A makes the following offer to her students: “For anyone who completes this series of
exercises by Friday, I’ll provide you with a free beverage from Starbucks.” Student B completes the
exercises by Friday but does not notify the Professor until Monday. Does B’s completion of the
exercises by Friday constitute acceptance of A’s offer? What if, on the same facts, the professor had
stipulated in her offer that the students had to not only complete the series by Friday but also had to
notify her that they had done so by Friday?

Unilateral offer. Yes. First Q: Communication wasn’t required. Second Q: the offer required it-student
hasn’t accepted it as hasn’t completed all steps.

(c) Third exception to the requirement that acceptance be communicated: VERY TESTED!!!

1) Under the common law mailbox rule:


Acceptance by mail is effective by dispatch so long as the acceptance is properly posted.

a) The mailbox rule applies only to acceptances and not to any other communication, so for
rejections, counter offers etc – it has to be received to be effective.
b) Consequences of the rule:
i) The first consequence of the mailbox rule:
offeror may not revoke once acceptance has been dispatched.

EXAMPLE:
While B was considering A’s offer, A had a change of heart and sent B a revocation, but B
dispatched his acceptance before receiving A’s revocation.
ii) The second consequence of the mailbox rule:
once offeree dispatches his acceptance, the parties have a binding contract and the offeree
may not withdraw acceptance.
iii) The third consequence of the mailbox rule:
offeror is bound even if acceptance is lost in transit.

c) A couple of nuances:
i) What of means of transmission other than USPS? (postal service)
the mailbox rule applies to any means of transmission that involves a foreseeable delay
between dispatch and receipt.

ii) Mailbox rule as a default rule:

alfrid.houston@gmail.com page 14/95


the offeror is a master of the offer – applies unless the offer applies the offer provides
otherwise.

HYPOTHETICAL

A’s offer to B stipulates that A “must hear from B by the close of business Thursday” in order for B’s
acceptance to be effective. B dispatches acceptance of the offer on Wednesday, but it doesn’t reach A
until Friday. Is B’s acceptance effective?

No – the offer said: “hear: I must hear, I must receive”. So no contract.

2) In a situation where the parties simultaneously dispatch identical offers:


the dispatch of the offers is not legally relevant. Offers are only effective to create the power of
acceptance upon receipt.

3) The Mailbox Rule and Option Contracts


a) the majority and restatement position: the mailbox rule doesn’t apply. Acceptance is only
effective upon receipt.
4) Hard Case: What happens when an offeree dispatches two responses to an offer, the first purporting
to reject the offer and the second purporting to accept it? – MBE tested!
a) The mail-box rule does not govern:
if the rejection is mailed first. So, whatever is received first by the offeror.
b) The parties’ obligations will depend on which of the offerees’ communications reaches the
offeror first.
i) If the acceptance reaches the offeror first:
There is a binding contract.
ii) If the rejection reaches the offeror first:
There is not a binding contract.

D. Acceptance and the UCC


1. The UCC’s Rejection of Common Law: Case #1—Acceptance by Seller’s Shipment of Nonconforming Goods
a. How might a seller accept a buyer’s offer?
(1) Under the UCC, a seller can accept a buyer’s offer to purchase goods for prompt or current shipment in one
of three ways:
(a) First way:
A promise to ship goods in conformance to the terms of the offer.

(b) Second way:


Prompt or current shipment of the goods in conformity of the terms of the offer.

alfrid.houston@gmail.com page 15/95


(c) Third way:
Shipping non-conforming goods.

EXCEPTION:
If the seller sends a non-conforming shipment as accommodation it constitutes a counter offer (this is a
counter ofer)

EXAMPLE:
Buyer orders 1,000 widgets from seller for immediate delivery. Seller responds by shipping 800 widgets
with an accompanying notice to the buyer explaining that the seller did not have adequate inventory to
ship 1,000 widgets and was thus shipping 800 widgets as an accommodation to the buyer in light of the
buyer’s urgent need.
1) Paradoxically, absent accommodation language, the seller’s shipment of nonconforming goods
does two things at once:
a) accepts the offer, forming a binding contract
b) breaches the contract (i.e. parties have to work it out).

2. The UCC’s Rejection of Common Law: Case #2—The Battle of the Forms
a. UCC Section 2-207 deals with the commercial setting in which parties transact business via the use of pre-printed
forms, and the difficulties created by application of the common law mirror image rule to such settings.
b. A so-called “battle of the forms” may occur when the terms of the form used by the seller do not match the terms
of the form used by the buyer.
(1) Note: though it is typically the buyer who “offers” to buy goods, these rules apply regardless of whether
buyer or seller is the offeror.
c. The ”battle of the forms” scenario problematically implicates two common law rules.
(1) The mirror image rule:
Under this common law rule, an acknowledgment of an orer form with diff terms will not create a
contract but the UCC changes this.

(2) The last shot doctrine:


Under the common law rule, the last shot fired controls the terms of the contract. The UCC
changes this too.

d. The UCC Solution to the “Battle of the Forms”


(1) Unless acceptance is expressly made conditional on assent to the additional or different terms (a
“conditional acceptance,” discussed later), the nonconforming acceptance will operate as an effective
acceptance of the offer, thus forming a contract.

EXAMPLE:
Buyer sends Seller a purchase order for 1,000 widgets at the advertised price of $10 each. Seller
sends Buyer an Acknowledgment of Order form that promises delivery of the widgets at the stated
price, but also contains boilerplate language that negates warranties and limits remedies in the
event of breach. Seller’s form will operate as acceptance of Buyer’s offer and create a binding
contract despite the presence of terms that vary from Buyer’s purchase order.
(a) “Dickered” Terms versus “Boilerplate” Terms
1) “Dickered” Terms
a) transaction-specific terms , identifying the goods to be sold, the quantity and price. These
terms have to be mirrored.
alfrid.houston@gmail.com page 16/95
2) Boilerplate” Terms
a) standard forms that appear in party’s forms, less likely to match.

e. Does Nonconforming Acceptance Affect the Terms of the Contract?


(1) Critical factual consideration:
Everything turns on the identity of the parties

(2) Transaction Involving a Consumer


(a) the additional / different terms are mere proposals for addition and they are no part of the contract
unless the offeror expressly agrees to them.

(3) Transaction Where Both Parties Are Merchants – more likely to be tested
(a) Critical Distinction: “Additional” versus “Different” Terms
1) “Additional” terms:
Those that address a new topic not addressed by the offer

2) “Different” terms:
Terms which purport to change an existing term of the offer.

(b) Effect of “additional” terms:


They automatically become part of the contract, except in 3 circumstances.

1) Exception #1:
The offer expressly limits acceptance to its own terms.

EXAMPLE:
“This order expressly limits acceptance to the terms stated herein.”
2) Exception #2:
Offeror objects to the additional terms within a reasonable time.

EXAMPLE:
This could be accomplished through language to the effect of: “We do not accept the binding
arbitration provision set forth in your Acknowledgment of Order.”
3) Exception #3: most tested
If the additional terms would materially alter the contract.

a) Definition of “material alteration”:


terms that would result in surprise or hardship if incorporated without the express awareness of
the other party. Minor terms are only supposed to get in automatically.

b) Examples of clauses that would materially alter the contract include:


i) Warranty disclaimers
ii) Clauses that materially shorten the deadline for raising complaints

iii) clauses that change usages of trade or past courses of dealing.


alfrid.houston@gmail.com page 17/95
EXAMPLE:
Buyer sends Seller a purchase order for 1,000 widgets at the advertised price of $10 each.
Seller sends Buyer an Acknowledgment of Order that promises delivery of the widgets at
the stated price and includes boilerplate language that negates all warranties and requires
payment for the order to be complete within 30 days of delivery, which is standard in the
widget trade.

(c) Effect of “Different” Terms


1) Majority rule/“knockout rule”: apply the majority rule unless told otherwise.
The different terms of each communication dealing with the same topic are knocked out and
omitted from the contract, so will need to negotiate.

EXAMPLE:
Buyer’s purchase order contains a choice of law provision stating that California law will govern
disputes arising from the transaction, and Seller’s order acknowledgment states that New York law
will govern. Under the majority rule, neither provision is part of the parties’ contract, and if the
parties desire a choice of law provision, they will have to negotiate one from scratch.
2) Minority rule:
The different terms of the acceptance will be treated as mere proposals for addition – the B has to
expressly agree.

EXAMPLE:
On the same facts as immediately above, but under the minority rule, the buyer’s California law
provision would govern the contract unless she expressly agreed to the seller’s contrary provision.

(4) Written Confirmations


(a) Parties sometimes contract orally and then send follow-up written confirmations, which may contain
terms that are additional or different than the original oral deal. As is the case for a contract formed by
an offer followed by a nonconforming acceptance, the treatment of the additional or different terms
depends on the identity of the parties.
1) Transactions not between merchants:
The terms in the written confirmation that are additional or different are mere proposals for addition.
They have to accept them.

2) In transactions between merchants:


a) Additional terms are automatically part of the contract unless:
i) materially alter the contract
ii) the receiving party objects to them in a reasonable time.

b) Different terms are:


mere proposals, the other party is free to accept or reject.

c) If both parties send written confirmations containing conflicting terms:


The knock out rule applies and neither term is in the contract.

f. Conditional Acceptance
alfrid.houston@gmail.com page 18/95
(1) Under the UCC, a definite and seasonable expression of acceptance will operate as an acceptance even
though it states additional or different terms unless acceptance is expressly made conditional on assent to
the additional or different terms” [§ 2-207(1)].

(2) In other words, if the acceptance is made expressly conditional on assent to the additional or different terms:
Then non-conforming acceptance will not be effective to form a contract.
(a) In this situation, no contract is formed by the writings until the offeror expressly assents to the additional
or different terms.

(3) What Constitutes a Conditional Acceptance?


(a) Majority rule:
Clear and conspicuous language that tracks the language of 2-207.
(b) Minority rule:
The response must clearly communicate that the offeree is unwilling to proceed unless and until the
offeror agrees to the new terms.

EXAMPLE:
Buyer sends Seller a purchase order for 1,000 widgets at the advertised price of $10 each. Seller
sends Buyer an Acknowledgment of Order form that promises delivery of the widgets at the stated
price, but also contains boilerplate language that negates warranties and limits remedies in the event of
breach. Seller’s form also contains the following language in large bold font: “Our acceptance of your
order is expressly conditional on your assent to the additional or different terms that appear in this
acknowledgment.” Under the majority rule, this exchange of forms does not create a contract; we have
an offer and a counter-offer. Under the minority rule, this exchange of forms would create a contract.

EXAMPLE:
Buyer sends Seller a purchase order for 1,000 widgets at the advertised price of $10 each. Seller
responds to Buyer’s order with an email saying, “We are in receipt of your order for 1,000 widgets at
the advertised price of $10 each, and we will be happy to fill it. However, before we can do so, we must
require you to agree to the following terms.” The message then sets forth terms negating warranties
and limiting remedies and concludes, “Upon receipt of your reply to this message stating your
agreement to these terms, we will immediately ship your order.” Under both the majority rule and
minority rule, this exchange of forms would not amount to a contract. Under both the majority rule and
minority rule, the Seller’s response would constitute a conditional acceptance.

g. Other Situations Governed by the Battle of the Forms Rules


(1) Contracts Formed by Conduct
(a) The UCC provides that the parties’ conduct in recognizing the existence of a contract is sufficient to
establish a contract even though their writings do not otherwise establish a contract.

EXAMPLE:
Buyer sends order to Seller, Seller responds with a conditional acceptance containing a negation of
warranties, and the parties have no further communications. Seller nevertheless ships the ordered
goods, and Buyer accepts and pays for them. Although the parties’ writings do not form a contract—
because Seller sent a conditional acceptance—a binding contract is formed by their conduct.

(b) If a contract is established in this way, the terms of the contract will be:
1) the terms on which the writings of the parties agree (i.e. terms in common)
(price, quantity, subject matter etc)

alfrid.houston@gmail.com page 19/95


2) together with:
Default terms provided by the UCC.
a) Note: The express terms in the parties’ communications which do not match or agree are
omitted.

(2) Transactions without Pre-Printed Forms


(a) 2 207 still applies to transactions even with no pre-printed forms.

(3) Shrink-Wrap Contracts


(a) The courts are divided on whether the rules of Section 2-207 apply to so-called shrink-wrap contracts
(i.e., agreements that consumers find once they open the boxes or packaging containing the goods).
(b) Most courts apply 2-207: (Majority)
In consumer transactions, these transactions are mere proposals which the consumers can accept or
reject.
(c) Other courts do not apply 2-207: (Minority)
If the consumer retains and uses the goods, they have accepted the offer on the merchant’s terms.

Question 1
On January 1, Trump left a message on Writer’s answering machine offering to hire Writer to create a training workshop on
Real Estate for Trump. On January 5, after Writer had received Trump’s message but before he’d had the opportunity to
return his call, Trump notified Writer that it was giving the contract to Writer’s competitor. Trump terminated the offer to
Writer.
Does Writer have any legal recourse against Trump?
A) No, because no contract was formed.
B) No, because consideration was not specified.
C) No, because an answering machine message does not meet the form requirements for an offer.
D) Yes, as Trump’s action was against the spirit of fair dealing in the common law.

Question 2
The faculty of a privately run law school posted a notice in the student lounge area where it could be viewed by all students
in the law school. The notice announced that, in an effort to encourage scholarship, the faculty was offering an additional
$1,000 prize to any student of the school who entered and won the “International Law in a Changing World” contest. The
contest was run by an outside organization which had no affiliation with the law school or the faculty.
In October, Hudson, a student at the school, saw the notice. She went straight to work on her paper for the competition. She also
left a note on the main counter of the Dean’s office which said, “I, Susan Hudson, accept the offer of the faculty for an additional
$1,000 prize in the ‘International Law in a Changing World” contest. Hudson did not know it, but the note was accidentally placed in
her file in the academic advisor’s office.
In mid-January, the original faculty notice was removed and a new notice was posted announcing that the faculty was
regretfully withdrawing its offer. On February 1, Hudson submitted her paper to the Dean’s office, which in turn passed it on
to the contest examiners. On April 3, it was announced that Hudson’s paper had won first prize. When she requested the
faculty to pay the additional reward, they refused.
Was the offer effectively revoked as to Hudson?
A) It was effectively revoked when the faculty’s revocation notice was posted in mid-January.
B) It was effectively revoked only if Hudson was aware of the mid-January removal of the announcement and its
replacement by the new notice prior to submitting her paper.
C) It was not effectively revoked because she had relied on the offer prior to February 1st. promisorry estoppel –
financial, losing opportunity etc
alfrid.houston@gmail.com page 20/95
D) It was not effectively revoked since the offer became irrevocable after a reasonable passage of time.

RECAP QUESTIONS:

Please take a few minutes and answer the following questions. Try to answer these questions without looking at your Bar
Notes. If you must go back to your notes, try rewriting the answer in your own words. This exercise will help improve your
memory retention of these concepts.

1. Describe a bilateral contract and a unilateral contract.

2. What are the two requirements of acceptance?

3. Explain the Mailbox Rule.

4. What is the UCC solution to the “Battle of the Forms”?

IV. CONSIDERATION AND PROMISSORY ESTOPPEL

A. Consideration and Bargain


1. The Bargain Theory of Consideration
a. The general rule in American contract law is:
that a promise is unenforceable unless supported by consideration.

b. The promise has some value and must be exchanged for something else of value, such as:
a counter promise or performance, given to the promisor by the promise as a quid pro quo for making the
promise.

2. One recurring consideration problem: tested very frequently.


a. Occurring often (though not exclusively) in family settings, the gratuitous promise is:
promise to make a gift. Such a promise lacks consideration and is generally unenforceable.

EXAMPLE:
A invites his newly widowed sister-in-law, who lives 60 miles from A’s farm, to come and live in his guest house
without charge so she’ll have a safe place to raise her family, explaining that he is eager to help her in her time of
dire need. Because the sister-in-law neither promised nor gave anything in return for A’s promise, A’s promise is
gratuitous and unenforceable.

3. Legal synonyms for “want of consideration”: i.e. no consideration.


a. lack of consideration
b. no consideration
c. not supported by consideration
d. insufficient consideration

4. Distinguishing “Want of Consideration” from “Failure of Consideration”!!!

alfrid.houston@gmail.com page 21/95


a. “Want of Consideration”:
the absence of a bargained for exchange (0 consideration)
b. “Failure of consideration”:
a party’s failure to perform in accordance of his promise (i.e. it’s a breach).

EXAMPLE:
If A promises to deliver a horse to B in exchange for B’s promise of payment and A then fails to deliver the horse,
then there was a failure of consideration. However, we would not say that the consideration for B’s promise was
lacking, since A’s original promise of performance constituted consideration for B’s promise of payment.

5. “Bargained-for” Consideration and the “Detriment” Test


a. The majority/Restatement rule is that:
a promise is supported by consideration if its based on a bargained for exchange.

b. Regardless of the jurisdiction, the question of whether a particular performance by the promisee constitutes a
“detriment” is answered by the application of the so-called legal detriment test.
(1) Under this test, the question is:
Whether the promisee is doing smth he had a legal right not to do or is foregoing some activity he
had a legal right to engage in.

EXAMPLE:
Promoter promised musician $5,000 to play a concert at Carnegie Hall. Promoter then tried to back
out of the promise, claiming that “you would have done it for free.” – he could have stayed at home.
So it is a detriment
EXAMPLE:
Uncle promised nephew $5,000 if nephew would give up smoking and drinking. The uncle’s estate,
hoping to avoid this obligation, argued that the nephew didn’t incur a detriment because giving up
smoking and drinking was good for his health.

6. Inadequacy of Consideration – bad deal


a. This refers to the situation where:
someone claims he wasn’t paid enough consideration for the deal.

b. The modern rule:


the courts don’t police the equivalence or fairness of the exchange for the purpose of applying the consideration
doctrine.

EXAMPLE:
If A promises his birthright for a mess of pottage from B, B has nonetheless provided consideration for the promised
birthright no matter how meager the value of the pottage. In other words, the supposed inadequacy of consideration
is no defense to a breach of contract claim.
EXCEPTION:
In some jurisdictions, an element of securing a specific performance is showing that there was a fair exchange.

EXAMPLE:
If A promised to sell Blackacre to B for a meager sum and then refused to deliver, some courts would refuse to
grant B specific performance because of inadequate consideration and instead limit B’s remedy for breach to
money damages.
alfrid.houston@gmail.com page 22/95
c. Do not confuse “insufficient consideration” with “inadequate consideration.”
(1) Insufficient consideration:
No consideration, so no bargain.

EXAMPLE:
Where A promises his sister-in-law a place to raise her family and seeks nothing in return.
(2) Inadequate consideration:
Assumes there was consideration but not enough so deal was unfair – i.e. a bad deal. But courts
don’t police the fairness.

d. Modern courts do not police the fairness of bargains via the consideration doctrine.
(1) Instead, a party may be able to defeat the enforcement of an excessively one-sided bargain through:
The defense of unconscionability.

7. Illusory Promises
a. Explanation and rule:
promise to perform that leaves performance to the discretion of the performing party and will no constitute
consideration.

EXAMPLE:
Thus, if A says to B, “I’ll sell you as many widgets as you want to order within the next two weeks for $5 a piece,”
and B agrees to buy at that price as many widgets as he decides to order from A, B’s promise is illusory and will
not constitute consideration for A’s promise.

8. Gratuitous Promises-tested
a. Defined:
promise to make a gift. Such a promise is not legally enforceable.

b. Executed Gifts distinguished:


gratuitous transfers are legally binding and enforceable (i.e. you can’t ask it back).

EXAMPLE:
If I promise you a new car for your birthday, that would constitute a gratuitous promise and would be
unenforceable if I decline to follow through. However, if I give you a new car for your birthday, then the transfer
is legally binding, and I cannot later change my mind and take back the car.
(1) What does it take to “execute” a gift?
(a) intent to give a gift
(b) actual or symbolic delivery

c. Bargains Distinguished
(1) Recitals of Consideration
(a) The rule:
The requirement of consideration is not satisfied by false recital.

alfrid.houston@gmail.com page 23/95


EXAMPLE:
A wants to make a binding promise to give his young granddaughter $10,000 on her 21st birthday and
accordingly gives her a note that reads: “For value received, I promise you $10,000 upon your 21st
birthday.” If no value was received in exchange for the promise, the recital would not satisfy the
consideration requirement and the promise would be unenforceable.
(b) A minority of courts recognize an exception to this rule where:
Written option contracts contain a false recital of consideration.

d. The Problem of a Condition on a Gratuitous Promise-recurring Q


(1) What is a “condition on a gratuitous promise”?
A condition is something a promisee must do to avail herself of the promisor’s gift but it doesn’t
count as consideration. A condition to get the gift doesn’t count.

EXAMPLE:
A invites his newly widowed sister-in-law, who lives 60 miles away from his farm, to come and live in
his guest house without charge so she’ll have a safe place to raise her family, explaining that he is
eager to help her in her time of need. The promisee here won’t be able to take advantage of A’s offer
unless she moves her family the 60 miles to A’s farm. However, the move is merely a condition of
the gratuitous promise, not consideration.
(2) Distinguishing a “Condition on a Gratuitous Promise” from “Consideration”
(a) Factor #1:
Language of parties used
1) words suggesting benevolence rather than gift

(b) Factor #2:


context
1) in commercial context, gratuitous promises are rare – more common in the family and haritable
context,

(c) Factor #3:


Benefit to the promisor
1) where the promisee’s detriment confers no benefit to the promisor, more likely that it indicates a
condition, and not consideration.

HYPOTHETICAL

Sprawlmart advertises a grand opening celebration for its 10,000th store, promising $500 gift certificates
to each of the first 10 customers on opening day. Is this a gratuitous promise or a bargain? If the latter,
what is the consideration?

Commercial context; they want people at the store.

alfrid.houston@gmail.com page 24/95


B. Alternatives in the Absence of Consideration
1. Past or Moral Consideration
a. a promise in exchange for something already given or performed is not support of consideration. Each value must
induce the other .

EXAMPLE:
In Mills v. Wyman [20 Mass (3 Pick.) 207 (1825)], the court refused to enforce the father’s after-the-fact promise
to compensate a Good Samaritan for nursing his dying son, and the case would likely come out the same way
today.

b. The Exceptions to the “Past Consideration” Rule


(1) Exception #1:
A written promise to pay a debt barred by limitations is enforceable.

(2) Exception #2:


A written promise to pay a debt discharged by bankruptcy is enforceable.

c. Enforcing an Unsupported Promise Using the “Material Benefit” Test – restatement 86


(1) This test is supported by the Second Restatement but is used in only a minority of jurisdictions. It provides
that a promise made in recognition of a past benefit conferred will be enforceable as long as:
(a) the promisee conferred the benefit on the promisor and not on a third party

(b) the benefit is material

EXAMPLE:
A sees that B is in grave danger and heroically intervenes to save the latter, injuring himself in the
process. B gratefully promises to compensate A for his efforts. These are the facts of Webb v. McGowin
[27 Ala. App. 82 (1935)], where the court enforced the promise because A’s efforts bestowed a material
benefit (the saving of a life) on B.
EXAMPLE:
Because the Good Samaritan in Mills v. Wyman bestowed the benefit of nursing services on the
promisor’s son rather than on the promisor, the promise would not be enforceable.

(2) An Exception or Limitation on the “Material Benefit” Test – come up couple of times
(a) a promise made based on past benefits is not enforceable if benefits previously contracted for by the
promisor.

EXAMPLE:
Employer promises a retiring employee a pension worth half the latter’s salary in recognition of the latter’s
many years of hard and loyal work. Although the employee’s past work no doubt bestowed material
benefits on the employer, the work was done under contract (i.e., the employee was paid for it under the
terms of the employment agreement), and accordingly the employer’s promise is not enforceable.

NOTE: Remember, the material-benefit test is a minority rule and most courts would in these circumstances

alfrid.houston@gmail.com page 25/95


apply the “past consideration” doctrine and allow exceptions only for the statute of limitations and bankruptcy
cases.

2. Promissory Estoppel
a. A promisee that reasonably relies to his detriment on a gratuitous promise may be able to enforce that promise,
even though consideration is lacking.
b. There are four requirements that must be met in order for promissory estoppel to be available.
(1) Requirement #1:
A promise-has to be specifc

EXAMPLE:
Statements like “I’d like to give you some money to help you with college” would be considered too
vague to be an actual promise. Alternately, “I’ll give $25,000 toward your tuition this fall” would be
specific enough to qualify as a promise.

(2) Requirement #2:


Foreseeable reliance

EXAMPLE:
The widow’s move to her brother-in-law’s farm on the faith of his promise would have been
reasonably foreseeable to the brother-in-law at the time he made her the promise. However, if the
widow had purchased a high-end SUV to facilitate the move, this would likely not have been
foreseeable.
(3) Requirement #3:

Actual reliance

EXAMPLE:
If the widow had already decided to make the 60-mile move before her brother made her the promise,
then this would not be actual reliance.

(4) Requirement #4:


Injustice without enforcement
(a) There are several specific factors that courts use to analyze the “injustice” requirement in an exacting
way:
1) the string of proof of the other 3 requirements
2) the willfulness of the breach
3) the relative position / equities of the parties (at the time of enforcement)
4) the extent to which reliance was detrimental
5) the availability of alternatives short of enforcing the promise.

HYPOTHETICAL

alfrid.houston@gmail.com page 26/95


Uncle promises his nephew $8,000 to enable the latter to purchase a neighbor’s car for transportation to
school and an after-school job. In reliance on the promise, the nephew secures possession of the
vehicle and promises the neighbor the $8,000. If the uncle reneges on his promise to pay, is promissory
estoppel available if the neighbor is willing to accept the return of the car and release the nephew from
debt? – this wouldn’t be availability of alternative short of enforcing the promise-so wouldn’t be available.
What if the car has a market value far in excess of $8,000, enabling the nephew to sell the car, cover
the debt to his neighbor, and recoup any costs associated with the sale? – not available – as an
alternative is available.

RECAP QUESTIONS:

Please take a few minutes and answer the following questions. Try to answer these questions without looking at your Bar
Notes. If you must go back to your notes, try rewriting the answer in your own words. This exercise will help improve your
memory retention of these concepts.

1. Explain the difference between a gratuitous promise and an executed gift—are either or both considered binding?

2. What are the three factors used to distinguish a “Condition on a Gratuitous Promise” from “Consideration”?

3. What are the four requirements that must be met for promissory estoppel to be available?

alfrid.houston@gmail.com page 27/95


V. STATUTE OF FRAUDS-I.E. CERTAIN TYPES OF CONTRACTS HAVE TO BE IN WRITING

A. Introduction
1. In contract law, contracts generally need not be in writing; oral and written agreements are equally enforceable.
2. The Statute of Frauds is an exception to this general rule.

B. The Statute of Frauds in Operation


1. If a contract is governed by the Statute of Frauds, then:
It has to be evidenced in writing and signed by the party against enforcement is sought
2. Who has a Statute of Frauds defense?
A party who doesn’t have a sign contract which is governed by the statute of frauds.

C. Evaluating Statute of Frauds Issues

EXAMP TIP: To analyze Statute of Frauds issues, ask the following three questions in order.

1. Does the Agreement Fall within the Statute of Frauds?


a. The Statute of Frauds only governs certain categories of contracts agreements. For example:
contracts for the sale of land or guarantees
b. If the agreement does not fall under the Statute of Frauds:
then signed writing is not required to enforce the contract – oral is fine.

2. Is the Statute of Frauds Satisfied?


a. If an agreement is within the Statute of Frauds, then the key question is whether the Statute’s writing requirement
is met.
(1) To satisfy the Statute of Frauds:
in writing, signed by the parties against whom enforcement is sought

(2) If the Statute of Frauds is not satisfied:


The party seeking enforcement will lose, unless there alternative enforcement.

3. Is Alternative Enforcement Available?


a. the party seeking to enforce the contract may be able to use another theory of enforcement to protect its interest,
such as promissory estoppel or quasi requirement

D. Analyzing Problems Under the Statute of Frauds

EXAM TIP: Remember the list of contracts that are within the Statute of Frauds: MY LEGS

1. First Question: Is the Contract within the Statute of Frauds?


a. In most states, six categories of agreements are governed by the Statute of Frauds:
(1) marriage contracts (e.g prenuptial agreement)
(2) year – contracts not to be completed within one year of their making
alfrid.houston@gmail.com page 28/95
(3) sale of land
(4) executor – contracts of an executor or administrator to answer for a duty of a deceased.
(5) guarantee-suretyship
(6) contract for the sale of goods at a price of $500 or more

b. In the typical case, there is little difficulty in determining whether a particular contract falls under the Statute of
Frauds. However, some categories have specific limitations within them that narrow the reach of the Statute of
Frauds.
c. Contracts Not to Be Performed within a Year
(1) The year at issue under the one-year provision is measured from:
Making the contract rather than the date of beginning performance.

EXAMPLE:
In June of her first year of law school, Law Student enters an oral agreement with Law Firm to work
for the firm during June, July, and August of her second summer. Although the duration of the
contemplated performance is only three months, the performance will not be complete until 14
months after the making of the agreement. Accordingly, the contract is governed by the one-year
provision and a signed writing is required in order to secure enforcement.

(2) The question of whether a particular contract is to be performed within one year of the making thereof is
answered by determining:
Whether at the point of formation it is at all possible to complete the required performance within a
year’s time. If there is a chance it can be completed in less than a year, it doesn’t need to be in
writing.

EXAMPLE:
It is highly unlikely that the construction of an oil-producing facility in a war-ravaged country would
be completed within a year. But because the performance is possible within that time—even if
exceedingly unlikely—the agreement is not governed by the one-year provision and no signed
writing is required.

EXAMPLE:
One of the parties to an oral construction agreement attempts to back out 14 months into the project
and raises the Statute of Frauds as a defense. Because the prospect of performance is measured
from the point of making the contract (when completion within a year was possible) rather than from
the time of the dispute (when completion within the first year is clearly no longer possible), the
agreement is not governed by the one-year provision and no signed writing is required.

EXAMPLE:
A three-year exclusive representation agreement between a professional athlete and his agent
cannot be performed within a year—no matter how diligently the agent works. Accordingly, the
agreement is governed by the one-year provision and a signed writing is required.

(3) Frequently Tested Situations


(a) Contracts that can be breached or excused within a year of its formation:
It is irrelevant. What matters is not whether it can be breached within a year but whether it can be
performed in full by its terms in a year.

alfrid.houston@gmail.com page 29/95


(b) A lifetime or permanent contract of employment:
This is not governed by the one year provision because death is possible.

d. Land-Sale Contracts
(1) The land provision has been modified under some circumstances.
(2) The contract for future sale:
Governed by land provision and requires a signed writing

(a) The present conveyance of land promised for money:


That’s outside of the land provision of the statute of fraud.
(b) Real estate brokerage agreements:
Although land is involved, they are service contracts and so are not governed by the land provision of
the statute of fraud.

1) Some jurisdictions have resolved this by adding a separate category to their Statute of Frauds
specifically for brokerage agreements.

(3) Lease of real property:


Are contracts falling within the land provision as a leasehold is an interest in land.

(a) Other jurisdictions exclude short-term leases of a year or less from the Statute
of Frauds.

e. Guaranty/Suretyship Agreements
(1) There are two exceptions to the general rule that a promise to answer for the debt of a third party—a
suretyship, or guarantor agreement—is subject to the Statue of Frauds and must be in writing.
(a) Exception #1:
When the creditor discharges the original debtor from its obligation on the faith of a guarantee of a third
party to pay the debt.

(b) Exception #2:-frequently tested


The main purpose doctrine – the main purpose of the guarantor is to protect his own economic interest

HYPOTHETICAL

Joe Plumbing Co. secures a loan from Piggy Bank. N. Vestor, who owns a controlling interest in the plumbing
firm, makes an oral promise to the Bank to guarantee repayment of the loan. Is the guarantee within the Statute
of Frauds? – oral guarantee is to protect his own interest. What if Vestor is a shareholder in Joe Plumbing Co.
but does not own a controlling interest in the firm?-much less likely to satisfy the main purpose doctrine.

alfrid.houston@gmail.com page 30/95


2. Second Question: Is the Statute of Frauds Satisfied?
a. If a contract falls within the Statute of Frauds, then the general rule is:
the contract is unenforceable unless evidenced in writing signed by the party against whom enforcement is
sought.

b. The Writing Requirement


(1) no requirement that the parties put their entire agreement in writing. Rather, all that is necessary that the
writing be a memorandum of the agreement, which can be prepared before, During or after contract
formation.

(2) Under the common law, the following memoranda have been held to satisfy the writing requirement:
(a) a letter from one of the parties to a third party describing the agreement

(b) the written offer acceptance of which formed the contract

(c) a letter from one of the parties repudiating by so admitting the agreement.

EXAMPLE:
“Dear Tony, I regret to inform you that I cannot go through with the sale of my racehorse to you. Although the
$100,000 you agreed to pay is very generous, my accountant has advised me of the tax implications, which I
had not considered, and so I must regretfully bow out.”

(3) The memorandum need not document the transaction in detail. Only the following terms are required:
(a) the identity of the parties to the transaction
(b) the nature and subject matter of the contract
(c) the essential terms of the agreement – e.g. price and date for performance

(4) Required Description under the Land Provision


(a) Older case law suggests:
A formal legal description of the land
(b) More current decisions suggest:
Address or some other description of the property may suffice.

c. The Signature Requirement


(1) The actual full signature of the party against whom enforcement is sought is not necessary.

alfrid.houston@gmail.com page 31/95


(a) What is needed?
Any symbol or mark used with the intention to authenticate the writing

(b) Examples:
Initials, typed, stamped or pre pre-printed signature or letterhead.

d. “Tacking Together” Multiple Documents


(1) The writing need not be a single document. A party may satisfy the Statute by tacking together several
documents which, once combined, satisfy all the necessary requirements for the Statute of Frauds.
(a) If all documents are signed, or if a signed document incorporates unsigned documents by reference:
Then the statute of frauds is satisfied.
(b) If unsigned documents are not incorporated by reference in a signed document, “tacking together” the
signed and unsigned documents to satisfy the Statute of Frauds is nevertheless permissible if:
1) First requirement:
There must be at least one signed writing unambiguously establishing a contractual rship between
the parties.

2) Second requirement:
The signed and unsigned documents clearly refer to the same subject matter

3) Third requirement:
There must be clear and convincing evidence of acquiescence towards the unsigned document by
the party against whom enforcement is sought.

EXAMPLE:
A new employee is sent a hiring letter, which makes no mention of a promise of job security.
However, the notes from the negotiations that preceded the sending of the hiring letter do say that
the company promised two years of job security. These notes, which were prepared by the
secretary, are not signed, whereas the hiring letter is signed. In this case, the notes can be tacked
on to the signed document.

e. Performance
(1) The Statute of Frauds may be satisfied with respect to some of the categories of governed contracts via part
performance, instead of signed writing.

(2) Land Contracts


(a) Part performance requires a showing of any combination, or all three, of the following:
1) payment of all or part of the purchase price
2) taking up possession
3) making substantial improvements to the property

(3) One-Year Contracts


(a) If fully performed:
Then an oral contract not to be performed within 1 year becomes enforceable.

alfrid.houston@gmail.com page 32/95


(b) If only partially performed:
If an oral contract not to be performed within one year is not enforceable

(c) Quantum Meruit


1) restitution – possible work around available to the Partly performing party

(4) Sale of Goods Contracts


(a) the UCC statute of frauds may also be satisfied by part performance of a sale of goods contract.

3. Third Question: Is there an Alternative Basis for Enforcement?


a. If the contract is within the Statute of Frauds, but the Statute is not satisfied because the breaching party never
signed a written document, the aggrieved party may nevertheless be able to secure some protection for her
interests via an action for restitution or promissory estoppel.
(1) Recovery for Benefits Conferred (restitution; quasi contract)
(a) where one party bestows a benefit upon another in connection with an oral contract even if the contract
is barred by the statute of frauds, the agreed party may recover the value of the benefits conferred –
ALWAYS WILL COME IN TO HELP

(2) Promissory Estoppel - trickier


(a) this may be available for detrimental reliance for losses suffered on the faith of an oral contract that is
unenforceable

(b) Easy case:


A party to an oral contract within the statute promises to the other he will create a signed writing and
the party relies on this by not taking steps to satisfy the statute most cts will apply promissory estoppel if the
promising party doesn’t create such a writing

(c) Hard Case:


No specific assurances for such a writing are made but the reliances are on the underlying oral contract
instead

EXAMPLE:
An employee who leaves his current job and moves across the country based on an oral two-year
employment contract, only to be dismissed before he starts his new job.
1) The slim majority of courts hold:
Promissory estoppels is available in this circumstance If there is strong evidence of reliance

2) A substantial minority of courts hold:


The common law promissory estoppel doctrine is pre empted by the statue of fraud.

E. Analyzing Problems under the UCC Statute of Frauds (OL IV. E.)
1. First Question: Is the Contract within the UCC Statute of Frauds?
a. The UCC Statute of Frauds is Section 2-201, which by its terms governs:
agreement for the sale of goods for the price of 500 dollars or more

2. Second Question: Is the UCC Statute of Frauds Satisfied?


alfrid.houston@gmail.com page 33/95
a. There are five ways to satisfy the UCC Statute of Frauds:
(1) The first way:
Signed writing
(a) The easiest way to satisfy the UCC Statute of Frauds, a signed writing requires the following to be
satisfied:
1) Requirement #1:
A writing
a) What counts as a “writing” under the UCC?
Any intentional redaction to tangible form
b) Electronic communication:
majority of courts have concluded that electronic documentation satisfies the writing
requirement of the UCC

2) Requirement #2:
Signed by the party against whom enforcement is sought

a) What counts as a ”signature” under the UCC?


Any symbol executed or adopted by present intention to authenticate a writing

b) Most states have adopted the UETA (Uniform Electronic Transactions Act) under which
electronic signatures satisfy legal writing requirements.
3) Requirement #3:
It must be sufficient to indicate that a contract for sale has been made between the parties

a) For this relaxed requirement, all that is needed:


the writing afforded a basis for believing that the authored oral evidence rest on a real
transaction

EXAMPLE:
A enters an oral agreement with B to sell the latter a speedboat for $10,000. B writes a check
that contains A’s name as the payee; B’s name as payor; $10,000 as the amount; and the
word “speedboat” in the notation space. Although the writing does not use the word
“agreement” or “contract,” it is clearly “sufficient to indicate that a contract of sale has been
made” between A and B and would, therefore, satisfy the writing requirement of the UCC
Statute of Frauds against B, the signing party.

(b) Required term:


A quantity term to satisfy the statute of frauds and the contract is unenforceable beyond the quantity
stated in the writing

1) If there is no quantity term, the contract is altogether unenforceable, subject to two exceptions:
a) Exception #1:
where other language in the writing provides unambiguous basis for measuring quantity

b) Exception #2:
output or requirements contracts which satisfy the requirements under the UCC
alfrid.houston@gmail.com page 34/95
c) Apart from the quantity term, no other term is required in the writing. However, a writing can
have so few terms that it will no longer be “sufficient to indicate that a contract for sale has
been made between the parties.”

EXAMPLE:
A and B enter an oral agreement under the terms of which A is to sell B five tobacco barns for
a total of $5,000. After they shake hands, A takes out a piece of paper, writes “five barns,”
signs it, and gives it to B. If A subsequently backs out of the deal, he will have a successful
Statute of Frauds defense against B despite the fact he signed a writing with a quantity term,
because the writing, taken as a whole, is insufficient to indicate that a contract for sale has
been made between A and B.

(c) Written Offers


1) Under the UCC:
A written offer doesn’t satisfy the writing requirement as it is insufficient to show that a contract has
been made

2) But if the requirements of the UCC “Firm Offer” rule are met:
Then this writing will be fully enforceable against the one making the firm offer

(2) The second way to satisfy the UCC Statute of Frauds:


The merchant’s confirmation
(a) The UCC Statute of Frauds may be satisfied when two merchants enter an oral agreement and one of
them sends the other a written confirmation of the agreement.
1) In such a circumstance, the Statute is satisfied against the recipient merchant:
If he fails to object to the confirmation within the required time

(b) Requirements for a valid merchant’s confirmation:


1) First requirement:
The confirmation has to be sufficient against the sender

a) satisfied if signed and contains a quantity term

2) Second requirement:
Writing is in confirmation of the contract
3) Third requirement:
Time frame for sending it
a) confirmation must be sent within a reasonable time of formation

4) Fourth requirement:
The recipient’s reason to know
a) the confirmation must be based on a real agreement or discussion between the parties and
must be actually received

(c) Exception with the recipient’s objecting response:


alfrid.houston@gmail.com page 35/95
The recipient must send a written notice of objection within 10 days of receipt in order to keep the
statute of fraud defense

EXAMPLE:
A and B enter an oral agreement under the terms of which A is to sell B five tobacco barns for a total of
$5,000. The following day, B sends A a written confirmation of the agreement and two days later B
receives a signed reply that reads in pertinent part: “My understanding is that we don’t have a contract
between us, for as I mentioned I am still entertaining other offers for the barns. My apologies for any
confusion that may have been caused.” Because A’s reply objects to the confirmation’s contents, the
confirmation does not satisfy the UCC Statute of Frauds against A.

(d) The “nightmare” of the admitting memorandum:


An admitting memo satisfies the statute of fraud against the sender – eg. Not objecting but admitting

EXAMPLE:
A’s signed reply to B’s confirmation reads: “I regret that I can’t go through with the sale of my barns to
you. Although the $5,000 you agreed to pay is very generous, my accountant has advised me of tax
implications of the transaction that I hadn’t considered, and I must, therefore, bow out.” Because A’s
signed reply is itself sufficient to indicate that a contract for sale had been made between the parties—a
contract from which A is now seeking to escape—the reply satisfies the UCC Statute of Frauds against A.

(3) The third way to satisfy the UCC Statute of Frauds:


In court admission

(a) A third means of satisfying the UCC Statute of Frauds occurs when:
A party admits in his pleading, testimony or otherwise in court that a contact for sale was made

1) The contract is only enforceable:


Up to the quantity admitted

(4) The fourth way to satisfy the UCC Statute of Frauds:


Partial performance
(a) Despite the absence of a signed writing, an otherwise valid contract is enforceable with respect to:
1) goods for which payment has been made and accepted

2) goods which have been received and accepted

(b) Thus, the partial performance exception does not apply unless:
There are actions by both parties indicating that the contract exists

(c) Divisible versus Indivisible Goods


1) If the contract in question involves divisible goods:
The contract is only enforceable up to the quantity actually delivered and paid for
2) If the contract in question involves an indivisible good:
Partial payment renders the contract fully enforceable
alfrid.houston@gmail.com page 36/95
(5) The fifth way to satisfy the UCC Statute of Frauds:
Problem of specially manufactured goods

(a) The problematic scenario:


Specifically manufactured goods to specifications of a buyer who then backs out.

(b) Statute is satisfied where the following five elements are present:
1) the goods are to be specially manufactured for the buyer;
2) the goods are not suitable for sale to others in the ordinary course of seller’s business;
3) seller has substantially begun to manufacture, or made commitments to procure, the goods;
4) the actions undertaken to begin to manufacture or procure occurred under circumstances which
reasonably indicate that the goods are for the buyer; and
5) the actions undertaken to begin to manufacture or procure occurred before seller received notice of
buyer’s revocation.

(c) Boiling those elements down to two critical concerns:


1) can the manufacturer resale in the ordinary course of business?
2) what was the manufacturer’s detrimental reliance completely before notification of buyer’s
withdrawal.

3. Third Question: Is there an Alternative Basis for Enforcement?


a. If a party has relied to its detriment on an oral contract whose enforcement is barred by the UCC Statute of Frauds,
the aggrieved party may be able to secure a remedy via promissory estoppel.
(1) Majority of courts:
Promissory estoppels is available if a strong case is shown

(2) Minority of courts:


Express language of the UCC statute of frauds precludes promissory estoppel

(3) Construction contractors:

All courts have protected a general contractor via promissory estoppel even if the oral sub
contract of goods is for 500 dollars or more.

F. Limitations on the Statute of Frauds (OL IV. H.)


1. The only effect of a successful Statute of Frauds defense is to defeat enforcement of the contract against the non-
signing party. The contract may still be valid and enforceable for other purposes.
a. the oral contract may provide evidence of a legal claim apart from breach of contract

EXAMPLE:
A brings an action against C for tortious interference with A’s oral land contract with B. Although B would have a
successful Statute of Frauds defense in a breach of contract action brought by A, the Statute will be irrelevant in
A’s tortious interference claim against C.

alfrid.houston@gmail.com page 37/95


b. the oral contract may provide evidence establishing a defense to a legal claim apart from breach.

EXAMPLE:
Under an oral contract for the sale of A’s land to B, B takes possession. A reneges and brings a trespass action
against B. Although A would have a successful Statute of Frauds defense in a breach of contract action brought
by B, the Statute would be irrelevant to the validity of B’s defense to A’s trespass claim.
c. the oral contract may provide evidence of the value of the services already rendered

EXAMPLE:
A spends three months working for B under an oral two-year employment contract. If B fires A and successfully
raises a Statute of Frauds defense against enforcement of the oral contract, A may nonetheless be able to
recover from B the reasonable value of the services rendered in quantum meruit. Moreover, he may offer the
price agreed to in the oral contract as evidence of the value of the services rendered.

RECAP QUESTIONS:
Please take a few minutes and answer the following questions. Try to answer these questions without looking at your Bar
Notes. If you must go back to your notes, try rewriting the answer in your own words. This exercise will help improve your
memory retention of these concepts.

1. What are the six categories of agreements covered by the Statute of Frauds?

2. What are the five ways one can satisfy the UCC Statute of Frauds?

3. What is the effect of a valid Statute of Frauds defense?

alfrid.houston@gmail.com page 38/95


VI. GAP-FILLERS, INTERPRETATION, AND THE PAROL EVIDENCE RULE

A. In General
1. Parties to a contract ordinarily reach explicit and unambiguous agreements on the essential issues of the contract,
such as quantity and price. However, there are two problems that arise.
a. gaps in the agreement that leave certain things unaddressed – eg when payment made

b. ambiguities in the agreement concerning issues parties tried to address.

B. Filling in the Gaps with Default Provisions


1. The metaphor of the “default rule”:
Judges use the default rules to fill in gaps when the contract doesn’t address the matter but parties are free to set
them aside by contracting otherwise.

2. UCC Default Rules for the Sales of Goods


a. Implied Warranties
(1) Warranty of Title
(a) For the sale of all goods, there is an implied warranty of:
1) good title to the goods
2) rightful transfer of the goods
3) no one liens are attached to this goods
(b) This warranty can only be excluded or modified by:
1) either specific language
2) circumstances which give the buyer reason to know that the seller does not claim unencumbered
title

(2) Warranty of Merchantability


(a) The warranty guarantees that
The goods are fit for the ordinary purposes for which those goods will be used

(b) only applies if the seller is a merchant


(c) The warranty of merchantability will be displaced by:
1) by specific mention of the word “merchantability” and conspicuousness if the disclaimer is in writing

2) any other language or circumstances that would be reasonably understood by the buyer to exclude
that warranty, e.g. “as is” “patent defects”

(3) Warranty of Fitness for a Particular Purpose-you don’t have to be merchant here
(a) This warranty grants that:
The goods being sold are fit for the particular purpose for which he buyer intends to use them

(b) The warranty only applies where, at the time of contracting, the seller has good reason to know:

alfrid.houston@gmail.com page 39/95


1) the particular purpose for which the goods are required

2) the buyer is relying on the Seller skill or judgment to select or furnish reasonable goods
(you don’t have to be a merchant here)

(c) The warranty may be negated:


1) where the disclaimer is in writing, clear and conspicuous

2) the goods have patent defects which are easily detectable

b. Express Warranties
(1) The UCC also provides that a contract can create express warranties. Express warranties by the seller that
the goods will conform to some standard arise whenever the seller expressly or impliedly makes them as
part of the basis of the bargain in the following ways:
(a) any affirmation of fact or promise
(b) any description of the goods
(c) any sample or model

(2) The seller need not use the words “warrant” or “guarantee” in order to create an express warranty.
(a) However:
A mere affirmation of the value of the goods does not create a warranty, it is a mere puffing.

c. Missing Terms-tested regularly


(1) Under the UCC, where there are pertinent terms missing, the following default rules will fill in those missing
terms:
(a) The default rule for a missing price term is:
A reasonable price at the time established by the contract for delivery

(b) The default rule for missing time term (e.g., time/date of delivery or time/date for action taken under
the contract) is:
A reasonable time
(c) The default rule for a missing place of delivery term is:
The Seller’s place of business

3. Common Law Default Rules for Service and Employment Contracts


a. Missing Price Term
(1) If one party performs services at the request of another, but no price is discussed in advance, then the default
rule under common law will apply. The default rule for a missing price term is:
A reasonable value for the services rendered

b. Missing Duration Term


(1) In practically every jurisdiction, the default rule for a missing duration term in an employment contract is:
The employment at will rule – the employer can hire and the employee can quit at any time and
for any reason (i.e. no duration).

alfrid.houston@gmail.com page 40/95


(2) In a majority of jurisdictions:
Policy docs or oral or written assurances of job security made to an individual employee can take
you out of the default rule. You need to look out in the exam for duration in other sources, like
personnel book

EXAMPLE:
Statements made during a recruiting interview that make assurances of job security.
EXAMPLE:
Written assurances contained in a personnel handbook.

4. The Obligation of Good Faith and Fair Dealing


a. Another source of “gap-filling” is the obligation of good faith and fair dealing. Both the UCC and the Restatement
of common law impose an obligation of good faith in the performance and enforcement of contract.
(1) In general, “good faith” is defined as:
Honesty in fact
(2) In the case of a merchant, “good faith” means:
Honesty In fact and the observance of reasonable commercial standards of fair dealing and the
trade

b. The good-faith obligation will also operate to ensure good faith where the terms of the contract leave a critical
term, such as the price, satisfaction, or quantity, open to the determination of one party. You need to exercise
good faith when you are left with discretion.

(1) Open Price Term:


(a) If a contract leaves the price to be fixed by one of the parties, then:
The party must fix the price in good faith.

(2) Satisfaction Term


(a) If a contract contains a satisfaction clause or similar term, then:
The determination as to whether a party’s performance is complete must be exercised in good faith.

EXAMPLE:
A commercial developer and the seller of a piece of land enter into a contract. The developer places a
condition on his purchase of the land that the seller first acquires “satisfactory leases” for the future
occupancy of the land. The developer must determine in good faith whether or not the future leases are
“satisfactory.”

(3) Open Quantity Term


(a) This arises in two contexts.
1) An output contract:
Contract in which the buyer agrees to purchase all the seller’s output for a particular good

2) A requirements contract:
The Seller agrees to supply the B with all the Buyer’s reqs for particular goods

alfrid.houston@gmail.com page 41/95


(b) Under the UCC, the party entitled to determine the particular quantity of goods to be sold—either the
buyer demanding delivery of his requirements or the seller demanding purchase of her output—must
make that determination in good faith.

(c) In addition to the good-faith requirement, the UCC prohibits any unreasonably disproportionate demand
or tender, if there was either:
1) a stated estimate
2) past course of dealing

EXAMPLE:
Seller and Buyer are parties to a three-year contract obligating Seller to supply Buyer with the
latter’s upsidasium bearing requirements for Buyer’s turbojet engine manufacturing plant. The
market price for upsidasium suddenly skyrockets, and Buyer greatly increases its demand for
bearings with the intention of selling them at great profit to third parties. Seller is not obligated to fill
the order because Buyer’s requirements demand was made in bad faith.
EXAMPLE:
The upsidasium market remains stable, but because of a decline in the commercial aviation industry, the
demand for Buyer’s engines declines precipitously and accordingly Buyer reduces its monthly demand for
bearings by 90% in comparison with the previous 24 months. Although Buyer’s reduced demand was
made in good faith, it is unreasonably disproportionate to its prior requirements and accordingly violates
Section 2-306.

C. Interpreting Ambiguous Language


1. The problem/issue:
The parties have addressed the topic in contract but have done so ambiguously such that the resolution of the mater
has been left unclear.

2. Objective Meaning versus Subjective Meaning


a. The rule:
the objective meaning of language trumps the subjective

EXAMPLE:
Contractor and Homeowner enter a contract for renovations to Homeowner’s home. The contract incorporates by
reference “the specifications dated 1/11/07,” which had been prepared by Homeowner and were attached to the
contract before signing. Contractor mistakenly thinks that the specifications referenced were an earlier version
prepared by Contractor, and he signs the contract on that understanding. The specifications prepared by
Homeowner and referenced in the contract are binding on both parties.

b. The rule is subject to two exceptions:


(1) Exception #1:
One party has reason to know of the other party’s subjective understanding, the first party is
bound by that understanding.

EXAMPLE:
The facts being otherwise as stated in the previous example, just before the parties sign the
contract, Homeowner overhears Contractor tell his business partner that he was glad Homeowner
had agreed to Contractor’s version of the specifications. Because Homeowner knew of Contractor’s
understanding of the contractual terms, that understanding controls.

alfrid.houston@gmail.com page 42/95


(2) Exception #2:
At the time of contracting both parties have the same subjective understanding, then that
subjective understanding will control

EXAMPLE:
General contractor is building a bridge. He hires a paving subcontractor to do work at a price to be
calculated in accord with the dimensions of the “concrete surface of the bridge deck.” At the time of
contracting, both of the parties understand that the concrete work included the sides and bottom as
well as the “surface” of the bridge deck, and accordingly that interpretation would be the controlling
interpretation.

3. Contra Proferentem and the Doctrine of Reasonable Expectations


a. The Rule of Contra Proferentem
(1) If an ambiguous term is included in the contract, then:
In case of doubt it is construed against the drafter

(2) Types of contracts subject to this rule of construction:


All contracts drafted by one of the parties, regardless of bargaining power

b. The Doctrine of Reasonable Expectations


(1) Even unambiguous terms may be interpreted against the drafting party if they conflict with the reasonable
expectations of the other party
(2) Types of contracts subject to this doctrine:
Mainly insurance contracts. In principle can apply to any contract of adhesion (i.e. take it or leave
it).

(3) Under the doctrine, courts make a distinction between the dickered terms and the boilerplate terms.
(4) The doctrine only applies to defeat the enforceability of:
The boiler plate terms which are inconsistent with the reasonable expectations of the purchaser.

EXAMPLE:
Fertilizer Co. purchased from Allied Insurance Co., an insurance policy covering burglary with fine-
print boilerplate language defining “burglary” as requiring “visible marks on” or “physical damage
to” the exterior of the insured building. During the policy term, Fertilizer Co. was the victim of a theft
in which the perpetrators were able to force open a Plexiglas door without leaving marks or damage,
and Allied refused payment on the policy. Because the definition of “burglary” was a boilerplate
term inconsistent with the reasonable expectations of the insured, the court interpreted the policy to
cover the theft.

D. Trade Usage, Course of Dealing, and Course of Performance


1. Trade usage, course of dealing, and course of performance are extrinsic evidence that can be used to both fill gaps
and resolve ambiguities in contracts.
2. Definitions under the UCC:
a. A usage of trade is any practice or method of dealing having such regularity of observance in a place or trade as
to justify an expectation that it will be observed with respect to the transaction in question.
b. A course of dealing is a pattern of conduct concerning previous transactions between the parties that is fairly to
be regarded as establishing a common basis of understanding for interpreting their subsequent expressions and
alfrid.houston@gmail.com page 43/95
other conduct.
c. A course of performance is present when a particular contract involves repeated occasions for performance by
a party and the other party, with knowledge of the nature of the performance and opportunity for objection to it,
accepts the performance or acquiesces in it without objection.

3. Distinguishing admissible uses of such evidence from their inadmissible uses:


a. Admissible uses:
(1) to fill in gaps
(2) resolve ambiguities
b. Inadmissible uses:
to contradict the express terms of the parties’ contract
c. Note, however, that course of performance evidence is uniquely available to:
establish a waiver or modification of even the express terms of the particular contract

4. In the event of a conflict, course of performance prevails over course of dealing and usage of trade, and course of
dealing prevails over usage of trade.

EXAMPLE:
Trade Usage Can Fill in a Gap in the Terms of the Contract: A is one of many suppliers of upsidasium, and B is
one of A’s long-standing customers. A fills an order by B and demands immediate payment; B refuses. It is standard
practice in the upsidasium industry for purchasers to pay invoices upon delivery, and therefore B’s refusal to do so is
in violation of the parties’ contract.
EXAMPLE:
Course of Dealing Trumps Trade Usage: Same facts as the previous example except that in the past, A has regularly
given B 30 days to pay for deliveries. Because this course of dealing between the parties trumps the payment-upon-
delivery usage of trade, B has 30 days within which to make the required payment.

EXAMPLE:
Express Terms of the Contract Trump Both Course of Dealing and Trade Usage: Same facts as the previous
example except that, the parties’ contract contains a provision requiring payment within seven days of delivery.
Because the express terms of a contract override both course of dealing and usage of trade, B has seven days within
which to make payment despite the past practice of paying in 30 days and the industry practice of immediate
payment.

EXAMPLE:
Course of Performance Can Establish Waiver or Modification of Express Terms of the Contract: Same facts as
the previous example except that A and B are parties to an installment contract and B has paid 30 days after multiple
deliveries without objection by A. This evidence of the parties’ course of performance is admissible to establish that A
has waived its contractual right to payment within seven days of delivery and/or that the parties have mutually modified
that obligation.

E. The Parol Evidence Rule – external to the four corners of the contarct
1. Introduction
a. Another source of evidence shedding light on the meaning of an agreement is the negotiations of the parties
leading up to the execution of the written contract. The parol evidence rule governs efforts to introduce such
evidence.
b. The rule governs the admissibility of the following type of evidence:
oral and documentary evidence of negotiations and other communications between the parties that took place
prior to or contemporaneously with in execution of the contract
alfrid.houston@gmail.com page 44/95
2. Approaching a Parol Evidence Analysis
a. When a party seeks to introduce parol evidence, the court will determine the admissibility of the evidence based
on two questions:
(1) what is the purpose for which the evidence is being introduced
(2) does the evidence relate to a term or contract which is integrated?

3. Integration
a. While the first step in the analysis is to determine the purpose for which the evidence is being introduced, the
effects of these different purposes will depend upon the level of integration in the contract.

b. Partial versus Complete Integration


(1) When a particular contract is partially integrated, it means that:
The terms contained are meant to be the final expressions of the terms

(2) On the other hand, when a contract is completely integrated:


A complete and exclusive statement of all the terms

c. Determining Partial Integration and Complete Integration


(1) The question of whether the terms of a contract are partially integrated (i.e., final) and whether an integration is
complete (i.e., exhaustive) is for the judge, not the jury, to decide.
(2) In reaching decisions regarding the complete integration of a contract, the judge will rely on the following
evidence:
(a) Merger Clause!
1) The most important evidence that the parties intended their written agreement to represent an
exhaustive account of their contractual obligations is the presence in the contract of a merger
clause reciting:
That the writing contains the complete and entire agreement of the parties or other words to that
effect – complete integration

(b) Other Evidence


1) Additional factors which are important for determining whether a contract as completely integrated
include:
Detail of the contract provisions as well as the length of the agreement itself

4. Purposes for Which Evidence Is Introduced


a. The effect of the rule depends on the purpose for which the parol evidence is being introduced:
(1) to explain or interpret the terms of the written contract
(2) to supplement the terms of the written contract
(3) to contradict the terms of the written contract

b. Purpose #1: To Explain or Interpret Terms of a Written Contract


(1) The majority rule:
Parole evidence is always admissible for this purpose

alfrid.houston@gmail.com page 45/95


EXAMPLE:
A and B have a written contract under the terms of which A agrees to sell B “all the uncut timber on
my property.” Evidence of conversations between the parties during a visit to the timber site is
admissible to explain that “property” means A’s country cottage and not A’s home in the suburbs.

c. Purpose #2: To Supplement Terms of a Written Contract


(1) The general rule:
Admissible unless the contract is completely integrated (i.e. has a merger clause).

(2) A completely integrated contract is intended by the parties to represent a final as well as an exhaustive
account of the parties’ obligations.

EXAMPLE:
A and B had a written contract under the terms of which A agrees to sell B “all the uncut timber on my
property,” and the contract contained detailed terms with respect to the felling and transportation of the
timber as well as a merger clause stating that the contract was “a complete and exhaustive account of the
obligations between the parties.” Evidence of an oral agreement between the parties at the time of signing
the contract that A would pay to have the trees inspected for termites before cutting would be admissible to
supplement the agreement only if the court concluded that the contract was partially (rather than completely)
integrated. Some courts would consider the merger clause to be conclusive on the question of complete
integration and thus bar the evidence, but other courts would treat it as persuasive only, evaluate the
proffered evidence of the supplemental oral agreement, and admit it if they concluded that the parties did not
intend the previously drafted merger clause to bar it despite their subsequent signatures on the contract.

(3) UCC Distinction for Sale of Goods Cases


(a) trade usage, course of dealing and course of performance can supplement even a completely
integrated agreement (so with a merger clause you can also bring this in).

EXAMPLE:
Same facts as in the previous example, except A and B are logging companies and B offers evidence
that sellers doing business in the commercial lumber trade invariably pay to have the trees inspected
for termites before cutting. This evidence would be admissible even if the court concluded that the
contract was completely integrated because under the UCC uncut timber equals “goods” and usage of
trade evidence is admissible to supplement a written agreement.

d. Purpose #3: To Contradict Terms of a Written Contract


(1) The general rule:
Parole evidence is not admissible for this purpose if the terms in question are integrated (you
don’t need complete integration, even partial integration suffices).

(2) Integrated terms are terms intended by the parties to represent their final agreement on the subjects in
question.

EXAMPLE:
A and B have a written contract under the terms of which A agrees to sell B “all the uncut timber on my
property.” Evidence of an exchange of letters between the parties in which they agreed that their agreement
would not include the shady trees in the immediate vicinity of A’s country cottage would contradict the written
expression—i.e., that A had agreed to sell B “all the uncut timber on my property”—and would therefore be
admissible only if the court concluded that the quoted provision not intended by the parties to be the final word
with respect to which timber would be cut (i.e., that the writing was not integrated). In making that

alfrid.houston@gmail.com page 46/95


determination, the court is free to consider the exchange of letters and other extrinsic evidence.

5. When the Parol Evidence Rule Will Not Apply


a. The parol evidence rule will not apply to the following types of evidence:

(1) Subsequent Agreements


(a) The parol evidence rule only applies to oral or written communications made prior to or contemporaneously with
the execution of a written agreement. This means that it will not apply to:
Subsequent agreements entered into after the execution of the written document.

EXAMPLE:
Buyer and Seller enter into a written agreement for the sale of 1,000 widgets. Two months before delivery is
to be made, the parties orally agree to modify the agreement and specify the goods as nonstandard type-Y
widgets, with Buyer agreeing to pay an additional cost. Evidence of this modification would not fall within the
parol evidence rule, as it was made subsequent to the execution of the written agreement.

(2) Collateral Agreements


(a) parole evidence rule will not affect agreements between the parties that are entirely distinct from the
written agreement in the contract at issue.

EXAMPLE:
A and B execute a detailed written contract, complete with a merger clause, by which A agrees to sell
A’s car to B. As part of the transaction, the parties orally agree that B may park the automobile in A’s
garage for one year, paying $25 per month. Despite the completely integrated writing, either A or B
may adduce evidence of the parking arrangement under the collateral agreement rule.

(3) Attack on the Validity of the Written Agreement


(a) The parol evidence rule only applies if there’s a valid written agreement. As a result, the parol evidence
rule will not bar efforts to prove:
That the written agreement is invalid or unenforceable

(b) A party can assert such a claim in one of the following ways:
1) Failure of an Oral Condition Precedent to the Agreement
a) evidence that the parties orally agreed to a conditions precedence to the contract taking effect
will not be barred

EXAMPLE:
The parties to a written contract for the sale of artwork orally agree that the sale will not take
effect unless and until a certificate of authenticity is issued by a named art expert. Because
authentication is a condition precedent to the sale, the parol evidence rule will not bar proof of
the oral agreement.

2) Absence of Consideration
a) evidence that a written agreement lacks consideration or that there was a false recital of
consideration is not barred.

3) Mistake or Duress
alfrid.houston@gmail.com page 47/95
a) evidence of mistake of duress is not barred by the parole evidence rule

4) Fraud
a) The majority rule is:
parole evidence of fraud is always admissible
b) The minority rule is:
such evidence is admissible unless the contract provides that the parties made no reps and
relied on no reps during the course in this agreement

5) Reformation
a) The parol evidence rule is also inapplicable where a party to a written agreement alleges facts
entitling him to reformation of the agreement.
b) For the plaintiff to obtain reformation, it must be shown:
i) there was an antecedent valid agreement (a real contract between parties)
ii) incorrectly reflected in the writing by mistake or fraud
iii) proof of these elements is established by clear and convincing evidence

RECAP QUESTIONS:

Please take a few minutes and answer the following questions. Try to answer these questions without looking at your Bar
Notes. If you must go back to your notes, try rewriting the answer in your own words. This exercise will help improve your
memory retention of these concepts.

1. Under the UCC, where there are pertinent terms missing, what are the default rules used to fill a missing price term, missing
time term (e.g., time/date of delivery or time/date for action taken under the contract), and a missing place of delivery term?

2. What is the parol evidence rule? What two questions will courts ask when determining whether to admit parol evidence?

3. When will the parol evidence rule apply?

4. When will the parol evidence rule not apply?

alfrid.houston@gmail.com page 48/95


VII. PERFORMANCE, MODIFICATION, AND EXCUSE

A. Obligations under the UCC – comes up from time to time


1. In General
a. The seller’s obligation:
is to transfer and deliver the goods
b. The buyer’s obligation:
to accept and pay for the goods

2. Carrier Cases versus Non-Carrier Cases-frequently tested. When does the risk of loss transfer
a. Carrier cases explained:
where the parties to a sales of good contract agree to use a common carrier

b. Non-carrier cases explained:


where the partis to a sales of goods contract do not agree to use such a common carrier

c. In non-carrier cases, when does the risk of loss transfer? – not v. often tested
(1) If seller is not a merchant:
The risk of loss passes to the B upon tender of delivery (i.e ready to deliver)

(2) If seller is a merchant:


Risk of loss passes only when the goods are physically in the B’s possession.

d. In carrier cases, when does the risk of loss transfer?


(1) Critical distinction:
(a) Shipment contract:
Contract where the S promises to turn the goods over to the carrier
(b) Destination contract:
Contract where the S promises to tender delivery at a particular destination point

(2) The default rule:


If The contract is silent, the contract is a shipment contract

(3) Regarding Risk of Loss


(a) Shipment contract:
The risk of loss passes to the buyer when the goods are delivered to the carrier
1) Identified as “FOB Seller.”
(b) Destination contract:
The risk of losses passes to the buyer when the goods are tendered at the destination contact specified
in the contract

1) Identified as “FOB Buyer.”


alfrid.houston@gmail.com page 49/95
B. Modification
1. At common law, modification of an existing contract was difficult. Contemporary rules allowing for modification under
the UCC and the common law are more flexible.

2. Modification at Common Law


a. The Preexisting Duty Rule
(1) a promise to increase compensation for duties already owed under an existing contract is an unenforceable
modification because there is no consideration.

EXAMPLE:
The captain of a fishing vessel promised to pay $100 to each of the sailors for their work on a fishing voyage.
Midway through the voyage, the sailors threatened to cease work unless they were promised an additional $50
each, and the captain reluctantly agreed. At the end of the voyage, the captain paid each of them the originally
promised $100 but refused to pay the $50 increase, so the sailors sued to recover the additional amount.
Because the sailors were already obliged to perform the work in question under the terms of their original
contract with the captain, his promise of an additional $50 is unenforceable under the pre-existing duty rule.

b. Exceptions to the Preexisting Duty Rule


(1) Mutual Modification
(a) A promise to increase compensation under an existing contract is enforceable as a mutual modification
if:
1) both parties agree to a performance that is different from the one required by the original contract

2) the difference of performance is not a mere pretence of a newly formed bargain

EXAMPLE:
During the course of the fishing voyage, the ship’s cook takes ill, and the captain instructs the youngest
of the sailors to perform the cook’s duties in addition to his fishing duties. The sailor refuses to perform
the work unless the captain promises him an additional $50 in compensation, and the captain reluctantly
agrees. The promise of additional compensation is enforceable as a “mutual modification” of the original
contract.

(2) Unforeseen Circumstances


(a) The preexisting duty rule will not apply if:
A promise of increased compensation is given in exchange for a promised performance that has been
rendered substantially more burdensome than reasonably anticipated at formation

EXAMPLE:
The fishing nets provided by the captain turn out to be defective and thus increase the workload of the
sailors in a manner substantially in excess of what was reasonably contemplated under the original
contract. The sailors threaten to cease the fishing unless they are promised an additional $50 each, and
the captain reluctantly agrees. The promise is enforceable despite the pre-existing duty rule in view of
circumstances not reasonably anticipated by the parties at the time of contracting.

3. Modification and the Sales of Goods under the UCC


a. In this context, the UCC specifically displaces the common law.
alfrid.houston@gmail.com page 50/95
(1) The UCC provides that:
An agreement modifying an existing contract for the sale of goods needs no consideration as long
as there is good faith

EXAMPLE:
A manufactures upsidaisium bearings and has a three-year contract to supply B with the bearings
the latter uses to make turbo-jet engines for commercial aircraft. Because of an unexpected world
shortage of upsidaisium, the price of the metal A uses to manufacture the bearings triples over a
short period, and A advises B that it can’t continue supplying the bearings unless B agrees to
increase the price to the point that A will break even on the supply contract. B reluctantly agrees. Is
the modification enforceable?

(2) Even when the modification is supported by consideration, the UCC provides that:
A modification is unenforceable if the appearance of the mutual bargain is merely a pre text to
hide a bad faith change of terms

EXAMPLE:
The facts being otherwise as stated in the previous example, A promises to ship the bearings to B in
blue cartons in exchange for B’s agreement to the higher price. Although the blue-box promise
might technically constitute consideration for B’s agreement to pay the new market price, the
modification is unenforceable for want of good faith.

C. Excusing Performance due to Faulty Assumptions


1. A party entering a contract makes many assumptions about the present and future. When such assumptions turn out
to be faulty, the parties may be excused under the doctrines of mistake, impossibility, impracticability, and frustration
of purpose.

2. Faulty Assumptions regarding Present Facts: Mistake


a. Requirement of materiality:
the mistake in fact must significantly impact the value of the transaction.

b. Unilateral versus Mutual Mistake


(1) Unilateral Mistake
(a) Defined:
One party is mistaken

(b) The rule:


A single party mistaken about present material facts is not excused.
1) Exception #1:
Other party knew or had reason to know of the party’s mistake

EXAMPLE:
A agrees to sell B a cow, which A knows to be barren, and, as the parties are writing up and signing
the agreement, B asks A a series of questions about the care and feeding of pregnant cows.
Because A has reason to know that B is mistaken with respect to the cow’s capacity to bear calves,
B’s obligation to purchase the cow is excused on account of unilateral mistake.
alfrid.houston@gmail.com page 51/95
2) Exception #2:
Where there has been a serious or unconscionable error-a minority rule (mainly clerical error)

a) This exception to the rule for unilateral mistake won’t apply if:
i) the error was caused by extreme negligence on the part of the party making the error

ii) the other party relied on the clerical error


b) This exception is a minority rule (available principally to general contractors, who face a
peculiar dilemma that we’ve previously discussed.)

(2) Mutual Mistake


(a) Defined:
Both parties have mutually made mistakes.

(b) Under the rules of mutual mistake, the contract will be voidable by the disadvantaged party where three
requirements are met:
1) the mistaken assumption must relate to a material fact

2) the mistake must be made by both parties


3) the disadvantaged party did not bear the risk of mistake under the agreement

EXAMPLE:
A agrees to sell B a cow at beef cow prices because at the time of contracting, both parties were
under the assumption that the cow was barren. A short time later, the cow was discovered to be
with calf, which greatly increased her resale value. The contract is voidable at the option of A.

EXAMPLE:
A agrees to sell B a cow at beef cow prices. At the time of contracting, both parties share the mistaken
impression that the cow is barren. B tells A that he is still going to try to breed the cow in any case. In
this case, A bears the risk of B’s efforts succeeding, because if B succeeds, it will prove the parties’
assumption that the cow was barren wrong. So A is accordingly bound by the contract even if the cow
turns out to be fertile.

3. Faulty Assumptions regarding Future Facts: Impossibility, Impracticability, and Frustration of Purpose
a. Impossibility
(1) The doctrine of impossibility excuses both parties from their obligations under a contract if the performance
has been rendered impossible by events occurring after the contract was formed.
(a) Requirement #1:
The impossibility must be objective-tested frequently on the distinction between objective and
subjective
1) Objective impossibility occurs:
When the performance under the contract literally impossible for anyone because of circumstances
under the control of the parties.

EXAMPLE:

alfrid.houston@gmail.com page 52/95


X promises to sell Y his horse, but the horse dies before X can deliver the horse.
2) Subjective impossibility occurs:
When the performance of the contract becomes impossible because of failure or fault on behalf of
the performing contract

EXAMPLE:
A party fails to have enough money to either make a promised payment or to obtain the
components required for the production of a promised product.
3) In what circumstances are we likely to find objective impossibility?
a) Circumstance #1:
when the subject matter of the contract is destroyed

EXAMPLE:
Buyer promises to buy Farmer’s 2009 wheat crop, the entirety of which is destroyed just before
harvest by wheat blight. The Farmer’s performance is excused on the basis of impossibility.
b) Circumstance #2:
when a personal services contract and the performing party has died or becomes
incapacitated.

c) Circumstance #3:
supervening law has rendered performance legally impermissible

(b) Requirement #2:


The contingency that creates the impossibility was not known to the parties at the time making the
contact

1) contingency rose after formation and was not anticipated by the parties.

(2) Note: Impossibility, as an excuse to performance of a contractual obligation, is available under both the
Common Law and the UCC.

b. Impracticability
(1) Courts are reluctant to excuse performance for any reason other than impossibility. However, under the
doctrine of impracticability, a promisor may be excused from performance where the following two
elements are proven:
(a) the contingency causing the impracticability was unforeseen

(b) the increase in the cost or burden of performance would be far beyond what either party anticipated

(2) Parameters of Impracticability under the UCC


(a) Increased costs, and the rise or collapse of a market are viewed as business risks, the sorts of
contingencies that fixed-price contracts are expected to account for.
(b) UCC cases where impracticability has been found typically involve:
Shortages caused by war, embargo, local crop failure or unforeseen shut down of major sources of
supply.

NOTE: Impracticability, as an excuse to performance of a contractual obligation, is available under both the
alfrid.houston@gmail.com page 53/95
Common Law and the UCC.

c. Frustration of Purpose
(1) Where a contingency occurs that dramatically reduces the value of performance to the receiving party, the
doctrine of frustration of purpose may be available to excuse the receiving party from its contractual
obligations.

EXAMPLE:
The doctrine of frustration of contractual purpose has as its source the famous case of Krell v. Henry, which
involved the owner of a London flat with a “ringside” view of the forthcoming coronation parade who agreed
to lease the flat at premium prices to a lessee eager to witness the festivities. The parade was canceled
when the King became ill, and the lessee’s contractual obligations to the owner were excused on the ground
that going through with the rental agreement in the absence of its raison d’être “cannot reasonably be said
to have been in the contemplation of the parties at the date of the contract.”

(2) The Modern Test


(a) The contemporary version of this rule will discharge a party’s contractual obligations when the following
three requirements are met:
1) Requirement #1:
The party’s principal purpose in entering the contract is frustrated.

a) This means that the frustration of incidental or non-material purposes would not trigger the
excuse of frustration of purpose.
2) Requirement #2:
The frustration was substantial in nature.

EXAMPLE:
In a variation on Krell v. Henry, where the King did not become ill and the parade was not
cancelled, the town erected bleachers that only partially obstruct the parade view from the
apartment.
3) Requirement #3:
The non-occurrence of the event precipitating frustration must have been a basic assumption of the
contract

(3) Frustration of Purpose and the Contractual Allocation of Risk


(a) Frustration of purpose (like mistake, impossibility and impracticability) operates as a default rule. If a contract
allocates the risk of the non-occurrence of an event that would frustrate a party’s purpose, the excuse of
frustration of purpose becomes unavailable.

EXAMPLE:
The facts otherwise being as they were in Krell v. Henry, assume that the parties’ written agreement contained
the following language: “The parties understand and agree that the flat owner will be arranging at great
expense to move himself and his family to the countryside in order to permit occupancy by lessee and
accordingly that lessee will be obligated under this agreement irrespective of whether the coronation parade
takes place during the lease period.” Since the contract allocated the risk of the contingency to the lessee, his
obligations under the contract were not excused for frustration of purpose when the parade was postponed.

EXAMPLE:
The facts otherwise being as they were in Krell v. Henry, assume that all of London was abuzz with news
of the new King’s possibly serious illness during the days before the parties entered into their agreement.
alfrid.houston@gmail.com page 54/95
Because a postponement of the coronation parade was therefore foreseeable, the lessee’s failure to
secure language in the parties’ agreement discharging his duties in the event of postponement constitutes
evidence that the lessee bore that risk and remained fully obligated under the contract when it came to
fruition.

NOTE: Frustration of purpose, as an excuse to performance of a contractual obligation, is available under both
the Common Law and the UCC.

D. Excusing Performance by Agreement of the Parties


1. Rescission
a. Is permissible where:
both parties to a contract are mid performance. Consideration is provided by each party’s discharge of the other’s
duties.

b. Note: The Statute of Frauds will not prevent an oral agreement of rescission that discharges unperformed duties
from being enforced unless rescission of a transfer of property is involved.
2. Accord and Satisfaction
a. The parties may make an accord, which is:
a contract under which the obligee promises to accept substitute of performance in satisfaction of the obligor’s
existing duty.

EXAMPLE:
Purchaser has paid builder in full for the construction of a house. Because of difficulties that neither party to the
contract could foresee, the builder is only able to partially complete the construction project. Purchaser agrees to
a lesser performance in return for a partial re-payment. This agreement is an accord. Builder then completes the
agreed-to substituted performance. This performance amounts to satisfaction of the accord.
b. Legal Effects
(1) Accord and satisfaction:
Satisfies the obligor’s original obligation.
(2) the accord alone:
Does not discharge the obligor’s duty but only suspends it.

c. Validity of Accord—Consideration Required – this gets tested.


(1) General contract law applies to accords, such that consideration is required.
(2) There may be sufficient consideration if:
(a) where the accord involves an agreement for partial or substituted performance:
The substitute of performance differs significantly from the original performance or its obligation is
doubtful
(b) where the accord involves an agreement for partial payment:
There must be a good faith or bona fide dispute about the amount owed.

E. Anticipatory Repudiation
1. Anticipatory repudiation may be established by:
a. a party’s definitive statement indicating it will commit a breach of contract; or
b. a party’s voluntary act that renders the party unable to perform its contractual obligations

alfrid.houston@gmail.com page 55/95


2. Adequate Assurance of Performance
a. If anticipatory repudiation cannot be established but there are:
reasonable grounds for insecurity, the insecure party may make a demand for adequate assurance of
performance.

EXAMPLE:
After making a contract with a manufacturer. A wholesaler become aware that the manufacturer’s employees are
out on strike. This gives reasonable grounds to confirm with the manufacturer that the contract will be performed.

b. Suspension of Performance by the Insecure Party


(1) Upon making a demand for assurances, a party with reasonable grounds for insecurity may suspend his
own performance so long as it is commercially reasonable to do so.
(2) The failure to respond with reasonable assurances constitutes a repudiation by the non-responding party.
This can occur where the other party:
(a) does not respond to the demand for assurances in a reasonable time, and 30 days under the UCC
(b) does not respond in a way that provides reasonable assurances.

3. Rights of the Aggrieved Party upon Repudiation


a. The aggrieved party’s rights upon this anticipatory breach:
(1) the aggrieved party can cancel the contract and terminate all rights and obligations under it
(2) the party can bring an action for damages or specific performance
(3) the party can ignore the repudiation and continue under the contract

4. Retraction of an Repudiation-sometimes tested over this.


a. A party who has made an anticipatory repudiation to the other party may retract the repudiation unless and until
the other party: does one of those three things
(1) acts in reliance
(2) accepts the repudiation by signaling this to the breaching party
(3) commencing a suit for damages or specific performance

F. Conditions (OL VI. C.)


1. In General
a. In some contracts, the obligation to perform is conditioned upon some event or action by the other party. Where
there are such conditions in a contract, obligations are triggered when that conditioning event or action occurs.
b. The law regarding conditions deals with two contractual problems:
(1) the order of parties’ performances – not often tested.
(2) the remedies where conditions in the contract have failed.

2. Terminology
a. Promissory versus Pure Conditions
(1) Promissory conditions:
Contact performance is conditioned on the occurrence of the promised performance by the other
party.

EXAMPLE:
alfrid.houston@gmail.com page 56/95
Consider a publication contract with the following language: “Publisher’s duty of payment is expressly
conditioned on Author’s timely delivery of the promised manuscript.” In this case, Author’s timely delivery of
the manuscript is a promissory condition on Publisher’s contractual obligation of payment.

(2) Pure conditions:


Contract performance is conditioned on the occurrence of events beyond the control of either
party.

EXAMPLE:
Consider a contract in which Skyblasters, Inc., agrees to provide a fireworks display for a holiday celebration
sponsored by the Village of Remulak, where the contract contains a “weather permitting” provision. In this
case, good weather is a pure condition on Skyblasters’s performance obligation.

b. Express versus Implied Conditions


(1) Express conditions:
Parties expressly include in provisions of the contract
(2) Implied conditions are:
Fiction employed by the court to deal with the potential effect of breaches of contract

3. Rules Governing the Order of Performance


a. See outline.

4. Rules Governing the Failure of a Condition


a. Failure of Condition under the Common Law
(1) The rights of the parties in the event of a failed condition depend on whether the condition is express or
implied.
(a) Failure of an Express Condition
1) The Rule:
The failure of an express condition will discharge the parties’ obligation to perform.

(b) Identifying Express Conditions


1) Look for clear language of the parties.

EXAMPLE:
“The party’s duty to perform is expressly conditioned on the promised performance.”
EXAMPLE:
“There is no obligation to proceed unless and until the following steps are taken.”
EXAMPLE:
“Payment is due upon completion”

(c) Excusing Failed Express Conditions


1) There are three situations in which the failure of the condition may be excused such that the
performance obligation of the party who stood to benefit from the condition is not discharged.
a) Situation #1:
waiver

alfrid.houston@gmail.com page 57/95


i) the party who has been discharged from performing by the failed condition may waive the
right to discharge and perform anyway

NOTE: Some courts treat this as a mid-term modification, which in a common law case requires consideration.

b) Situation #2:
bad faith conduct
i) when a party acts in bad faith so as to prevent the condition that party’s performance
obligation will not be discharged

EXAMPLE:
A landowner’s obligations to pay a contractor for the building of a structure are conditioned on the
issuance of a certificate of occupancy by the inspector of buildings. The landowner takes steps to
induce the inspector not to issue the certificate, the landowner is in bad faith interfering with the
condition and it will be excused. Landowner will owe the contractor even though the certificate has
not been issued.

EXAMPLE:
In a contract for the sale of real property, the contract is conditioned on the buyer securing
financing. If the buyer fails to apply for financing, this would be considered conduct in bad faith,
and the buyer would be obligated to pay.

c) Situation #3:
gross forfeiture
i) the courts will excuse a performance obligation so as to avoid a grossly disproportionate
loss to a party for a relatively minor infraction

(d) Implied Conditions


1) When the possibility of a breach is not addressed by an express condition, it is still a breach; however,
under the law of implied conditions, courts can treat that breach in one of two ways: either as a material
breach or as substantial performance.

2) Material Breach versus Substantial Performance


a) If the breach is serious enough, the court will treat the breach in the same way it would treat a
failure of an express condition. This is called:
material breach and the agreed party is generally discharged from its own performance
obligation

b) If the breach is less serious, the court will treat the party’s performance as “close enough,”
meaning that the party has rendered:
substantial performance, the agreed party will not be discharged from its own performance
obligations.

c) The Restatement of Contracts lists five factors that can help to distinguish between material
breach and substantial performance:
i) the extent to which the aggrieved party will be deprived of the benefit that she reasonably
expected to receive under the contract;

alfrid.houston@gmail.com page 58/95


ii) the extent to which the aggrieved party can adequately be compensated via damages for
the defective performance;
iii) the extent to which the breaching party will suffer forfeiture if a material breach is found;
iv) the likelihood that the breaching party will cure his failure; and
v) the extent to which the breach was willful or in bad faith rather than merely negligent or
innocent.

HYPOTHETICAL

Contractor builds a house for Landowner, and—unbeknownst to Contractor—the plumbing


subcontractor installs a brand of pipe that differs from the brand specified in the contract but is in every
important respect the same quality. It would be an extremely expensive and burdensome task for
Contractor to remove and replace the pipe. Is this a case of substantial performance or material breach?

Substantive performance

HYPOTHETICAL

Contractor builds a house for the Yuppies, a young couple who are foodies and love to entertain, and
accordingly their plans include a spacious kitchen. The Yuppies spend the year in Paris while the house
is under construction, and in their absence and without prior consultation, Contractor decides to deviate
from the plans and build a smaller kitchen, thus enabling Contractor to save significant construction
costs. Is this a case of substantial performance or material breach?

Material breach

(e) Failed Condition That Cannot Be Excused


1) Where a condition has failed and cannot be excused, there are other methods of enforcement
available to mitigate the consequences for the breaching party.

2) Divisibility of the Contract


a) The breaching party may be able to argue:
that the contract is divisible and only a part of it has been materially breached.

alfrid.houston@gmail.com page 59/95


b) The legal test for “divisibility” of the contract:
whether it is easily apportioned into agreed equivalents.

EXAMPLE:
A one-year cleaning service contract would be divisible if the services were provided and paid for on a
monthly basis, as it would be easy to apportion the larger exchange into “pairs of part performance”
that are “properly regarded as agreed equivalents”—i.e., a month’s worth of cleaning in exchange for
a month’s pay. To put it another way, the one-year contract is merely the sum of the 12 monthly
exchanges.

EXAMPLE:
A enters into a contract to build a house for B for $100,000, with progress payments of $5,000 due in
monthly installments during construction and a balloon payment upon the architect’s certificate of
satisfactory completion. The contract is not divisible, for the performances and the progress payments
are not “agreed equivalents,” and the whole is greater than the sum of the parts, since no reasonable
person would enter a free-standing contract for a month of construction work.

EXAM TIP: If the whole is merely the sum of its parts, it is divisible. If the whole is greater than the sum of the
parts, it is not divisible.

3) Quantum Meruit
a) Where a party failed to fulfill an express condition or is in material breach, he may still be able
to recover in quantum meruit:
i) the breaching party may recover the reasonable value of the benefits conferred

ii) such recovery will be reduced by the damages caused by its breach of contract

b. Failure of Condition under the UCC


(1) Express and Implied Conditions in Relationship to the Perfect Tender Rule
(a) Under the perfect tender rule, the terms of a contract for the sale of goods are enforced exactly. Every
contract term is thus treated as an express condition, and a breach by seller will relieve the payment
obligation of the buyer.
(b) The rule is that the seller is in breach:
If the goods fail in any respect to conform with the contract.

(c) If the seller fails to make perfect tender, the buyer has three courses of action available:

1) Reject the Goods


a) For a buyer to reject the goods, the buyer must:
exercise the right of rejection within a reasonable time after delivery and notify the seller

b) Once a buyer rejects the goods, the buyer may:


bring an action for damages against the seller on account of the imperfect tender unless cure
applies

c) If the buyer does not effectuate rejection in the manner specified above, then he has made a

alfrid.houston@gmail.com page 60/95


failed rejection, which:
is deemed as an acceptance of the goods by the B.

2) Accept the Goods


a) The buyer may accept the goods, despite the improper tender. Acceptance occurs when the
buyer has had a reasonable opportunity to inspect the goods and signifies acceptance either
through:
i) stating to the seller that the goods conform to the contract
ii) taking the goods despite their non-conformance
iii) failing to make an effective rejection
iv) taking any action inconsistent with the seller’s ownership of the goods.

b) What are the legal consequences of the buyer’s acceptance?


i) the buyer must:
pay the contract price for the goods
ii) the buyer may also:
seek damages for non-conformity so long as the seller has been seasonably notified.

iii) The buyer can revoke the acceptance if there is a non-conformity that substantially impairs
the value of the goods and if (1) the reason for the acceptance was that the buyer was
unaware of the non-conformity or (2) the seller assured the buyer that a known non-
conformity would be cured but the seller failed to do so.
Perfect tender applies only where you initially accept them.

3) Accept Part and Reject Part


a) The critical rule:
when accepting part and rejecting part, the B can do so only in terms of commercial units of
that good

(d) Work-Arounds for Breaching Sellers


1) The right to cure: (i.e. before the deadline)
If a seller makes a non conforming but times for performance remains under the contract the seller
may substitute performing goods.

a) Two requirements:
i) the seller must give the B seasonable notice of its intention to cure
ii) seller must make confirming delivery within the time specified in the contract

2) Reasonable Grounds to Believe Delivery Was Acceptable (past the deadline)


a) Two requirements:
i) S must give B seasonable notice of his intention to cure
ii) S must make a conforming delivery within a reasonable time

3) Proof of Reasonable Grounds by Seller


a) A seller may prove that he had reasonable grounds to believe that the buyer would accept
alfrid.houston@gmail.com page 61/95
nonconformity if he has evidence of:
i) express assurances to that effect by the buyer
ii) trade usage – course of dealing – course of performance evidence to that effect

(e) Imperfect Tenders in the Context of an Installment Contract. Installment contract is an exception
to the perfect tender rule
1) Under UCC, installment contracts are contracts that contemplate the delivery of goods in separate
lots to be separately accepted by the buyer.
2) When a particular installment occasions a non-conforming tender, there are three possibilities that
the UCC deals with:
a) If the non-conforming tender substantially impairs the value of the whole contract:
there is a breach of the whole contract and it can be cancelled
b) If the non-conforming tender substantially impairs the value only of this particular installment:
the B can reject the installment but cannot cancel the entire contract
c) If the non-conforming tender does not even substantially impair the value of this particular
installment:
minor non-conformity: the B must allow the S to cure the non-conformity within a reasonable
time.

APPLICATION

Question 3
A woman hired her neighbor to clean up her pool before her big July 4th pool party. The neighbor agreed to do the work for
$50. The day before the party, the woman noticed that her neighbor still had not started the work. She asked her neighbor if
the pool would be clean in time for the party. The neighbor replied, “I thought about it and realized $50 is too low a price for
the work involved. I don’t think I’m going to clean the pool.” The woman responded, “If you can clean the pool by tomorrow
morning, I’ll pay you an additional $25. And I won’t sue you in small claims court for your failure to perform your promise.”
The neighbor then cleaned the pool that day, but the woman refused to pay to him anything more than $50.
In a suit by the neighbor to recover the additional $25 promised by the woman, the neighbor will:
A) win, because he performed his part of the bargain.
B) win, because the second contract for $75 superseded the original $50 contract.
C) lose, because the $75 contract did not supersede the $50 contract.
D) lose, because he had a preexisting duty to clean the pool for $50.

APPLICATION

Question 4
In 1992, Daniel Gere saved the life of Charlene Mendenhall’s husband George, after he was involved in an accident. George
later rewrote his will to leave $5,000 to Gere for his actions. However, when George died in 1996, he left behind no property
except for an undivided interest in a piece of land held in tenancy by the entirety with Charlene. She had purchased the property
in 1983 with money she had inherited from her father’s will. After George died, Charlene signed and delivered to Gere a written
instrument which said, “In consideration of Daniel Gere’s having saved my husband George’s life and his agreement to make
no claims against my estate based on George’s will, I promise herewith to pay Daniel Gere the sum of $5,000.” Late last year,
Charlene Mendenhall died, and Gere filed a claim against her estate for $5,000. Hathaway, Charlene Mendenhall’s executor,
contested Gere’s claim, indicating that Gere’s instrument was not supported by sufficient consideration.
Gere’s strongest argument would be:
A) that Charlene benefited from Gere’s actions.
alfrid.houston@gmail.com page 62/95
B) that Charlene is bound by promissory estoppel.
C) that Charlene and Gere had reached an enforceable compromise agreement.
D) that Charlene had executed a binding unilateral agreement.

alfrid.houston@gmail.com page 63/95


RECAP QUESTIONS:

Please take a few minutes and answer the following questions. Try to answer these questions without looking at your Bar
Notes. If you must go back to your notes, try rewriting the answer in your own words. This exercise will help improve your
memory retention of these concepts.

1. When does the risk of loss transfer in carrier cases? In non-carrier cases?

2. What is the preexisting duty rule?

3. What is the doctrine of impracticability? How does it differ from impossibility?

4. What is the perfect tender rule? What three options are available to the buyer if this rule is violated?

alfrid.houston@gmail.com page 64/95


VIII. DEFENSES –

A. Introduction:
6 CATEGORIES, ALL OF WHICH ARE AVAILABLE TO A PARTY IN COMMON LAW AN UCC CASES

B. Incapacity
1. Infancy/Minors
a. Infancy is the time period before a person reaches the age of majority. An infant, commonly referred to as a
minor, is:
any person under the age of 18.
b. The modern rule:
minors may enter into a contract but the contract is voidable at the option of the minor.

(1) The special case of minors who are married or emancipated.


(a) The majority rule:
Minors still enjoy the power of avoidance

(b) The minority rule:


Marriage or emancipation eliminates the power of avoidance

c. Power of Avoidance (or Disaffirmance)


(1) Upon exercising the right to disaffirm a contract, the minor is obligated to return to the other party any goods
received under the contract.
(a) The majority rule is that the minor must return the goods if they are in his possession when he
disaffirms the contract, but:
He is not liable for any damage, wear and tear or any other depreciation of value of the goods.

(b) A contract between a minor and another party may be for something that cannot be returned, such as a
services contract or a lease. In such a case:
The minor is under no further obligation to compensate the other party

(c) In a minority of jurisdictions, a minor can only disaffirm a contract by making the other party whole. In
these jurisdictions, the minor would be liable for:
Depreciation, damages, as well as compensating the other party for the services received

d. Ratification-he wants to keep it


(1) once minor turns 18, they may expressly or impliedly ratify contacts in entering majority and thus bind itself
themselves to the obligations they may have otherwise disaffirmed.

e. Exceptions
(1) Necessaries-critical exception (eg
(a) A minor’s contract for necessaries:
Is voidable but the merchant always has a quasi contract right to recover the reasonable value of the
alfrid.houston@gmail.com page 65/95
goods or services
1) Necessaries are:
Food, clothing, shelter and medical care
a) Courts are split:
re education and automobiles

(2) Misrepresentation by Minor


(a) In a minority of jurisdictions, if a minor has misrepresented his age to the contracting party in order to
obtain the goods or services of the contract, he may be:
Equitably estopped from proving his real age in cort and thus prevented from invoking the defense of
infancy.

2. Mental Incompetence
a. Under modern rules, a person lacks capacity to contract if he was mentally incompetent at the time of contracting.
b. Requirements for Mental Incompetence
(1) If a party is adjudicated incompetent and a guardian is appointed, then this adjudication will be sufficient to
establish mental incompetence for contract cases.
(2) Absent prior adjudication of incompetence, then the defense will be available under the following
circumstances:

(a) Cognitive Defects


1) A person will be deemed mentally incompetent:
If he is unable to understand in a reasonable manner the nature and consequences of the
transaction.

EXAMPLE:
A person who is operating under the influence of delusions or hallucinations will be deemed
mentally incompetent based on cognitive defects.

(b) Volitional Defects


1) In many jurisdictions, mental incompetence can be established if:
a) a person unable to act in a reasonable manner in relation to the transaction, and

b) the other person has a reason to know of this condition

EXAMPLE:
A person who is a manic-depressive would be able to avoid a contract if he was in a manic
phase at the time of contract formation and if the other party had reason to know of that
person’s state based, for example, on his erratic behavior.

c. Legal Consequences of the Mental Incompetence Defense


(1) The legal consequences of a successful mental incompetence defense depend on whether or not the party
exercising this defense has been adjudicated mentally incompetent or not.
(a) If there’s been an adjudication of mental incompetence:
Contracts are altogether void
(b) If there’s been no such adjudication:

alfrid.houston@gmail.com page 66/95


Contract is voidable at the option of the incompetent party

(2) Possibility of Ratification


(a) a party who was mentally incompetent at the time of contract may expressly or impliedly ratify the
contract if he becomes competent later on

(3) The mentally incompetent party’s duty of restitution:


ii.e. if you avoid the contract. If the mentally incompetent exercises his right of avoidance and has
received a benefit under the contract, he is required to make the other party whole by paying
reasonable value of goods or services rendered.

EXCEPTION:
If the other party takes an unfair advantage of the mentally incompetent person by being aware of
his incompetence then the other party is entitled only to the return of benefits still in possession of the
mentally incompetent party

(4) In the case of a contract for necessaries:


The mentally incompetent can disaffirm the contract but the provider of the necessaries can
recover in quasi contract the reasonable value of services rendered

(a) An addition to the list of necessaries:


Legal representation in connection with incompetency proceedings

3. Incapacity defenses are available under both the common law and the UCC.

C. Misrepresentation
1. Types of misrepresentation
a. Fraudulent Misrepresentation
(1) Four elements:
(a) Element #1:
A misrepresentation
1) the D must have made an assertion which is inconsistent with existing facts

2) What would be sufficient to prove this element?


a) oral or written misrepresentation
b) fraudulent conduct such as concealment
c) a half truth

EXAMPLE:
A seller of real estate undertakes efforts to hide termite damage.
3) Misrepresentations are not:
a) broken promises, but misstatement of existing fact
b) opinions or guesses
4) But there are exceptions:
a) professional opinions are treated as representations of the facts in the matter

alfrid.houston@gmail.com page 67/95


EXAMPLE:
An automobile dealer who tells a buyer that he thinks the car runs pretty well, even though he
knows that the car does not run well, would be disguising a fact as an opinion. This would
count as a misrepresentation to the buyer.
EXAMPLE:
A certified mechanic who tells a buyer that he thinks the car runs pretty well is likely speaking
as an expert. We presume that professionals and experts, when speaking in their areas of
expertise, base their views on underlying facts. This can also count as a misrepresentation to
the buyer.

(b) Element #2:


State of mind
1) The requirement of scienter is satisfied if the defendant made the assertion either:
a) knowing it to be false
b) knowing he had no idea whether it was true or false (either one suffices)

EXAMPLE:
Auto dealer says, “this baby can go from zero to 70 in six seconds flat.” If he either knows that
the car can’t accelerate that quickly, or has no basis for knowing whether the car can
accelerate that quickly, scienter is present.

2) The “intent to mislead” requirement is satisfied if the defendant made the assertion either:
a) for the purpose of misleading the agreed party, or

b) knowing that there was a substantial likelihood that the agreed party will be misled

(c) Element #3:


Materiality of the misrepresentation (objective or subjective)

1) Objective Materiality and Subjective Materiality


a) Objective materiality explained:
where such an assertion is likely to induce a reasonable person to enter into a contract

EXAMPLE:
An assertion that a used car has been carefully inspected by an independent auto mechanic
would appeal to a reasonable person, and thus satisfy the objective materiality test.

b) Subjective materiality explained:


the party making the assertion had a reason to know that it was likely to induce the particular
party into the contract

EXAMPLE:
An assertion that the car was owned by Salman Rushdie when he was living incognito in the
U.S. would be material if made to an individual whom the seller knew to be a Rushdie
enthusiast.

alfrid.houston@gmail.com page 68/95


(d) Element #4:
Reasonable reliance on the misrepresentation
1) What would count as unreasonable reliance?

EXAMPLE:
Reliance would be unreasonable if the aggrieved party has independent knowledge or reason to
know that the statement in question is false.
EXAMPLE:
Reliance would be unreasonable if the aggrieved party has reason to believe that the statement in
question was made by a person who is unreliable.
EXAMPLE:
Reliance would be unreasonable if no reasonable person would have believed the assertion.
EXAMPLE:
Reliance would be unreasonable if the aggrieved party could have easily ascertained the truth by
cursory inspection of the goods.

b. Non-Fraudulent Misrepresentation
(1) Two types:
Negligent and innocent
(2) Four elements:
(a) Element #1:
Misrepresentation
(b) Element #2:
Materiality of the misrepresentation
(c) Element #3:
Reasonable reliance on the misrepresentation
(d) But the fourth element is different:
Don’t need scienter or intent to mislead
1) The state of mind requirement for negligent misrepresentation is:
The D would have known the assertion was false had he exercised reasonable care

2) The state of mind requirement for innocent misrepresentation is:


The D made an assertion not in accord with the existing facts

c. Fraudulent Nondisclosure
(1) The fraud here consists of:
The defendant’s silence when there was a duty to disclose

(2) Three elements:


(a) Element #1:
The non-disclosure was material to the contract
(b) Element #2:
Reasonable reliance on the non-disclosure
(c) Element #3:
alfrid.houston@gmail.com page 69/95
A duty of disclosure and a failure to fulfill it

(3) Although there is generally no duty of disclosure to trading partners, if a party is aware of material facts
that are unlikely to be discovered by the other party in the exercise of ordinary care and diligence, then
there will be a duty to disclose that information in these circumstances:
(a) where parties enjoy a relationship of trust and confidence

(b) where parties made an assertion which was true at the time but has since been rendered untrue by
intervening events

(c) the obligation of good faith will require that the party disclose the information

2. Remedies for Misrepresentation


a. What‘s available for all three types of misrepresentation?
(1) Use as a “shield” (avoidance):
All can be used as defenses to a breach of contract claim
(2) Use as a “sword” (rescission and reliance damages):
All can be used in a tort action for rescission of the contract and also for damages resulting from
reliance on the misrepresentation

b. What’s available to fraudulent and non-fraudulent misrepresentation but not fraudulent nondisclosure?
They may live with the contract and sue for the benefit of the bargain.

c. What’s available to fraudulent misrepresentation but not to the other two types of misrepresentation?
A party can seek punitive damages because it is an intentional tort

3. Defenses are equally applicable under common law and the UCC.

D. Duress
1. Under modern law, there are three elements to a defense of duress:
a. Element #1:
a threat
(1) Defined:
Manifestation of intent to inflict harm on a person made in words or conduct

EXAMPLE:
An example of conduct considered a threat is the scene in “The Godfather,” where the man wakes
up with a horse’s head in bed, which was placed there to induce him to sign the contract.

b. Element #2:
the threat is wrongful in nature
(1) Three types or circumstances:
(a) if what is threat is either crime or tort
(b) what is threat is either criminal prosecution or bad faith civil process

alfrid.houston@gmail.com page 70/95


(c) if what is threat is a bad faith breach of contract – economic duress.

EXAMPLE:
Sailors on a fishing voyage cease their work midway through the voyage and refuse to resume their
duties unless the captain agrees to give them a raise.

1) Distinguish a bad-faith threat of breach of contract from a good-faith demand:


If the demand is due to an increased burden caused by unanticipated circumstances then there is
no duress

EXAMPLE:
The sailors’ demand for additional compensation in response to an unanticipated expansion of their
workload due to faulty fishing nets would be a good-faith demand.

c. Element #3:
there must be no reasonable choice but to succumb to the threat
(1) This is seldom an issue when what is threatened is a crime or a tort, a criminal prosecution, or a bad faith
civil suit, for risking the eventuality of any of those is scarcely a “reasonable” choice.
(2) Hard cases often arise in cases involving “economic duress” (i.e., threatened breach of contract) and
commonly arise in these three situations:
(a) Situation #1:
When there are no adequate or reasonably priced substitutes for the services or goods threatened to
be withheld

EXAMPLE:
The ship’s captain is not in a position to find replacement sailors when already at sea, so no adequate
substitutes would be available.

(b) Situation #2:


When the threatened breach would cause the agreed party to breach his own contract

EXAMPLE:
A purchaser of upsidaisium bearings used in the manufacture of turbo-jet engines would, on
account of the breach, be forced to breach its contract with an aircraft manufacturer.

(c) Situation #3:


When the alternative of acquiescing to the threat and then suing for damages is inadequate to address
the harm caused (e.g. reputation).

EXAMPLE:
where the threatened breach would cause the aggrieved party to renege on its own commitments and
thus harm its reputation and opportunities for future business, damages would be inadequate as a
remedy.
d. A minority of jurisdictions add another element to the requirements for “economic duress”:
the agreed party must protest the threatened breach rather than acquiesce without complaint.

2. Legal Consequences for Duress

alfrid.houston@gmail.com page 71/95


a. Avoidance:
contracts made under physical compulsion are absolutely void; all other contarcts are voidable.

b. Restitution:
the agreed party is entitled to restitution of any benefits conferred as long as he returns any benefits received

3. The common law defense of duress can be used in cases arising under the UCC as a supplementary provision.
E. Undue Influence

EXAMPLE:
After being arrested for sodomy, a school teacher was released and arrived at his home late at night. When he arrived there, he
found his superintendent and principal, who demanded his immediate resignation, threatening otherwise to commence
termination proceedings and noting the likely unwanted publicity that would follow.
1. There are two elements to the defense of undue influence.
a. Element #1:
unfair persuasion used
(1) The test for this element involves a long list of factors: you don’t need all of them
(a) discussion of the transaction at an unusual or inappropriate time

(b) consummation of the transaction at an unusual place

(c) insistent demands that the transaction or business be finished immediately

(d) extreme emphasis on the consequences of delaying the transaction

(e) the use of multiple individuals persuading the target of persuasion

(f) absence of third party advisers to the target of persuasion

(g) statements that there is no time to consult advisors or attorneys

b. Element #2:
vulnerable party to persuasion
(1) A vulnerable party can be established if one of the following circumstances exists:
(a) Circumstance #1:
Mental infirmity is due to age or illness
(b) Circumstance #2:
Where the vulnerability is due to a recent trauma or event

(c) Circumstance #3:


Where there is a relationship of trust or confidence – eg relative, client attorney

2. Legal Consequences of Undue Influence


a. contract is voidable at the option of the agreed party

alfrid.houston@gmail.com page 72/95


b. the agreed party is entitled to restitution of any benefits conferred on the other party so long as he also returns
any benefits received

3. Like the other defenses, the claim of common law undue influence is available in cases arising under the UCC as a
supplementary provision.

F. Unconscionability
1. The Elements of the Unconscionability Defense
a. Element #1:
procedural unconscionability
(1) Defined:
Where the bargaining process created an absence of a meaningful choice
(2) In what circumstances will this be found?
(a) Circumstance #1:
Near miss cases

EXAMPLE:
A signatory is competent but, because of advanced age or language barriers, cannot understand the
terms of the contract and operates as if under mental incompetence or duress.

(b) Circumstance #2:


Absence of bargaining power
1) Examples include:
a) adhesion contracts – standard form contracts with take it or leave it provisions; or party
weakened by poverty or language barriers

(c) Circumstance #3:


Fine print terms

b. Element #2:
substantive unconscionability
(1) Defined:
Contract terms are unreasonably favorable to one party to the contract

(2) In what circumstances will this be found?


(a) Circumstance #1:
Grossly excessive price

EXAMPLE:
where a pay-over-time plan requires the consumer to pay a total sum that is many times the value of
the purchased goods; or where a bank charges an overdraft fee that is many times the bank’s actual
processing costs.

(b) Circumstance #2:


Grossly disproportional consequences for a minor breach

alfrid.houston@gmail.com page 73/95


EXAMPLE:
An add-on clause, pursuant to which a merchant is entitled to repossess multiple household items,
including furniture, bedding, and a stereo, when a consumer missed a payment, despite the fact that
the consumer had already paid nearly 80% of the monies owed for the various purchases.

(c) Circumstance #3:


Provisions binding one party but not the other

EXAMPLE:
an arbitration provision in an employment contract requires the employee but not the employer to
arbitrate any disputes, or permits the employer but not the employee to amend the provisions of the
agreement at will.

(d) Circumstance #4:


Provisions which are grossly unfair

EXAMPLE:
In Brower v. Gateway 2000, the court found unconscionable a binding arbitration clause that required
consumers pursuing disputes with a merchant to pay a non-refundable advance fee that exceeded the
purchase price of the goods, travel to a distant city for the proceeding; pay the merchant’s legal fees in
the event of a loss, and file all correspondence in a foreign country.

2. Legal Consequences of a Finding of Unconscionability


a. Upon a finding of unconscionability, the court may:
(1) refuse to enforce the contract
(2) excise the offending clause and enforce the remainder of the contract
(3) limit the application of the offending clause so as to avoid any unsconscionable result

G. Public Policy/illegality
1. Public policy may be raised as a defense to the enforcement of a contract in the fol lowing four contexts:
a. Context #1:
where the subject of the contract itself is specifically prohibited by law

EXAMPLE:
In most jurisdictions, a contract for prostitution, gambling, or bribery is illegal under the law and accordingly
unenforceable in court.

b. Context #2:
where the contarct is formed for the purpose of committing a crime

EXAMPLE:
A contract between an employer and a hired assassin would be a contract for the commission of a crime.

c. Context #3:
where the contarct performance would constitute a tort

EXAMPLE:

alfrid.houston@gmail.com page 74/95


A publicist being hired for the express purpose of spreading a defamatory story about a private citizen would be a
contract for the commission of a tort.

d. Context #4:
where the contract performance would violate certain values and freedoms designated by the state

EXAMPLE:
A contract that prohibits one party from marrying for an extended period of time would violate the public policy of
promoting free and consent-based marriages.

2. Sources of Public Policies


a. Legislation
(1) frequently the source of policy involved

b. Judicial Decisions
(1) Where the subject matter of the contract is not specifically prohibited under law, then judge-made public
policies may be invoked, which may include policies based on:
(a) moral and social values
(b) economic considerations
(c) protection of governmental processes and institutions

EXAMPLE:
A contract under which a private citizen agrees to vote a certain way in exchange for a year’s supply of
beer would violate the public policy of free and unrestricted exercise of the right to vote.

3. Operation of the Defense of Public Policy


a. The public policy defense is almost invariably invoked as a defense in an action by one of the parties seeking
enforcement of a contract against the other, and when it is successful:
the defendant wins, irrespective of whether he was the party promising to perform the pulic policy violation or the
party paying for it.

(1) The reason for this is that:


The courts will play no role in enforcing such a contract and will use the parties as it finds them

EXCEPTION:
(Not) In pari delicto exception:
Where one of the parties is much more egregiously in the wrong than the other, it may be
possible for the other party to secure restitution of benefits conferred on the guiltier party

b. A contract that is subject to the defense of public policy is:


voidable at the option of the defending party

4. Likely Contexts for Public Policy Defenses


a. Noncompete Agreements
(1) Many employment contracts as well as contracts for the sale of a business contain a provision that prevents

alfrid.houston@gmail.com page 75/95


one party from competing for a certain period of time or in a certain area.
(a) Although generally enforceable, such contracts may be deemed contrary to the public policy of
promoting a citizen’s freedom to work.
(b) Consider the extent to which the non-compete provision imposes an unreasonable geographical barrier
or duration upon the employee

b. Sales of Goods via Bribery


(1) If a contract for the sale of goods was based on bribery of one party by the other, then the victim of the bribe
has a public policy defense.

EXAMPLE:
Buyer sues seller for breach of contract when seller refuses to supply the specified quantity to buyer
at the agreed-upon price. Seller raises the defense of public policy, because after the fact Seller
learns that buyer bribed Seller’s agent with a car in order to secure the unreasonably favorable
contract terms.

c. Sales of Goods intended for Unlawful Use


(1) If a contract is for the sale of goods that the seller knows the buyer intends for an unlawful use, then the
public policy defense is available to defeat either an action by the seller seeking payment or an action by the
buyer seeking delivery.

d. Liability Limitation Provisions


(1) If the provisions of a contract would limit a party’s liability for tortious behavior by restricting the right of the
injured party to pursue claims against the reckless or intentional harms caused by the party, then such
provisions will not be upheld by a court because to do so would violate public policy.
(2) In contrast, provisions that limit liability for negligent tortious conduct are generally held to be
permissible.

e. Unlicensed Goods or Services


(1) If a contract is for unlicensed goods or services, then the contract is voidable and may be rendered
unenforceable by a public policy defense.

APPLICATION

Question 5
Abbott owns several acres of commercially zoned real estate. For several months, Costello has been trying to convince
Abbott to sell him a portion of the property for development as a shopping mall. One day during a golf game at the club,
Abbott orally agrees to sell Costello half of his land. On the back of his score card, Abbott scribbles in pencil the following
note: “June 1, 1999. I will sell to Costello one half of my property located at the corner of First Street and Grand Avenue.”
Abbott signs the note as well. Excited, Costello goes to see his bankers and obtains a promise of financing. Costello tenders
the purchase price to Abbott but Abbott rejects the money, saying that he has changed his mind. Costello brings an action for
specific performance. Abbott raised the defense of the Statute of Frauds, and Costello introduces the score card with
Abbott’s memorandum as evidence to support the contract. Abbott argues that the score card is an insufficient memorandum
of their agreement.

Which of the following statements best supports Abbott’s argument?


A) The memorandum was not signed by Costello.
B) The memorandum does not adequately identify the property to be sold. – It doesn’t for e.g. say which half-
alfrid.houston@gmail.com page 76/95
North/south etc.
C) The memorandum does not contain a covenant of title.
D) The memorandum did not set the date for performance.

APPLICATION

Question 6
Darren was tall for his age. He was the only student in his 7th grade class over six feet tall. One day, while walking home from
school, Darren and his friend Chris entered an automobile dealership. It quickly became apparent that the salesman thought
Darren was an adult, and Darren did not correct him. The dealership was offering a zero down special. Darren found a car he
really liked, and agreed to make the required $800 a month payments. Darren and Chris drove off the lot with the new car. Chris
was afraid that Darren would get in trouble, so he called the dealership and spoke with the salesman who had sold Darren the
car, informing him of Darren’s age. An hour later, Darren was driving his new car when he was involved in an accident which
heavily damaged the front of the car. Darren used the car’s built in cell phone to call the dealership. He informed them that he
was actually 12 years old and wished to get out of the contract.

Under contract law, Darren is


A) responsible for damage to the car, because he must return the merchandise in the same condition in which he
received it. – this is a minority rule.
B) responsible for damage to the car, because he willfully misrepresented his age. – minority rule. Didn’t represent his
age
C) not responsible for damage to the car, because Chris’s call notifying the dealership of Darren’s true age voided the
contract.-they are voided at the option of the minor
D) not responsible for damage to the car, because as a minor Darren can disaffirm the contract.-correct answer.

RECAP QUESTIONS:

Please take a few minutes and answer the following questions. Try to answer these questions without looking at your Bar
Notes. If you must go back to your notes, try rewriting the answer in your own words. This exercise will help improve your
memory retention of these concepts.

1. With regard to the defense of incapacity, what is the rule regarding infants and minors?

2. Describe the three types of misrepresentation.

3. What are the elements of duress?

alfrid.houston@gmail.com page 77/95


IX. REMEDIES

A. Remedies for Breach of Contract Generally


1. Monetary Damages at Common Law
a. A party aggrieved by a breach of contract may be able to recover damages calculated to protect that party’s
expectation interest, reliance interest, or restitutionary interest. A party may:
elect only one of these remedies

b. First type:
expectation damages
(1) This is the default award courts will grant aggrieved parties.
(2) An award of expectation damages means that the aggrieved party will be entitled to the amount:
That will restore him to the position he would have been in had the contract been fully performed

EXAMPLE:
A patient with a hand injury contracts with his doctor for hand surgery. The doctor promises him a
“100% perfect” hand, but instead the surgery makes the hand worse. The expectation interest of the
patient is measured by the difference between the value of the promised “100% perfect” hand and
the value of the hand worsened by surgery.

(3) Calculating the Monetary Award-you want to know this formula


(a) The formula:

Loss of value of the breaching party’s performance


+ any incidental and consequential costs generated by the breach
– any payments received from the breaching party
– any costs saved as a result of the breach

= Expectation Damages of the Aggrieved Party

EXAMPLE:
General contractor repudiates its contract with Subcontractor when Subcontractor is halfway finished
with its work under the subcontract. Subcontractor is entitled to seek from the breaching General
contractor an amount equal to the contract price or the unrealized value of General’s promised
performance, plus costs associated with storing, insuring, and/or returning materials and equipment
secured by Subcontractor in the course of contractual performance (i.e., incidental costs generated by
the breach) or the incidental costs generated by the breach; minus any progress payments already
made by General contractor; minus money Subcontractor may have saved on salaries and equipment
rental by not having to complete performance.

(4) Limitations on the Right to Recover Expectation Damages-very often tested


(a) The aggrieved party may not be able to recover the full amount of expectation damages in the following
situations:

alfrid.houston@gmail.com page 78/95


1) Situation #1:
The cost of performance greatly exceeds the market value of the performance

EXAMPLE:
In the infamous case of Peevyhouse v. Garland Coal & Mining Co., landowners entered into a
contract with a coal-mining firm permitting the latter to strip-mine their farmland but requiring the firm
to restore the land to its original state once the mining project was complete. When the firm breached
by refusing to do the restoration work, there were two ways to measure the resulting expectation
damages: the cost of performance (or the costs that would be incurred in an effort to restore the land
as promised in the contract), or the net increase in the market value of the property that would be
realized if the restorative work had been completed as promised. Because restoration would have
cost the mining firm $29,000 but increased the market value of the land by only $300, the court limited
the landowners’ recovery to the latter figure.
a) Many courts would award cost of performance despite the disparity with market value on the
view that expectation damages should be calculated on the basis of the value of performance
to the injured party himself and not on its value to some hypothetical reasonable person or on
some market.
2) Situation #2:
Cannot be calculated with reasonable certainty

a) A recurring example of a case where uncertainty in awarding expectation damages exists is:
a new business with no profit history
3) Situation #3:
Damages are unforeseeable (Hadley v Baxendale)

HYPOTHETICAL

Supplier agrees to deliver a quantity of window glass to Store for $1,000 on a certain date. Store needs
the glass to fill its depleted inventory. Supplier fails to deliver as promised because the market price of
the quantity of glass has risen to $1,200 and Supplier prefers to sell to a third party at the higher price.
Unbeknownst to Supplier, Store had, in the meantime, entered a contract to sell the entire quantity of
glass for $1,500 to a business services firm for use in the many properties it manages, and Store is
unable to deliver because of Supplier’s breach. To what general damages is Store entitled? (just the
200$)Suppose Store’s buyer explained at the time of contracting that it needed timely delivery of the
glass in question because it had already committed to selling the quantity in question to a third party.
How would this change the analysis? (full 500$) Suppose the buyer of the glass is not a store but a
glazing contractor that needs the glass in question to install windows in a new commercial building, and it
will be required to pay liquidated damages to the general contractor if it fails to perform in a timely
fashion. How would this change the analysis

Just the $200

alfrid.houston@gmail.com page 79/95


4) Situation #4:
Where damages can be mitigated

EXAMPLE:
The liability of a breaching buyer of produce would be reduced if the seller permitted the produce to
rot after breach rather than selling it to third parties; but if the seller is forced by the breach into a
“fire sale” because of the short shelf life of the produce in question, then the breaching buyer will be
fully liable for expectation damages — that is, for any difference between what the seller nets from
the “fire sale” and what it stood to earn but for the broken contract.

EXAMPLE:
The liability of an employer is reduced if an employee dismissed in breach of contract takes an
extended vacation rather than seeking other work. But the aggrieved employee need only make
reasonable efforts to secure a position that is reasonably equivalent to the job lost, and accordingly
she need not accept substitute work when it: is in a different field; offers significantly lower pay or less
desirable terms and conditions of employment than those of the lost job; would entail more
burdensome responsibilities than those of the lost job; or would damage the aggrieved party’s career
prospects. (e.g., an A-list movie star would not be required to mitigate damages for a film producer’s
breach of contract by taking a lesser role in a B-grade film).

EXAMPLE:
The liability of a breaching landowner to a construction contractor hired to erect a structure on the
property is reduced where the contractor continues the building project after the landowner has
repudiated the contract. But the liability of a buyer of goods ordered for special manufacture would not
be reduced if the seller continues the manufacturing process after breach in order to finish the goods for
possible sale to third parties.

c. Second type of Monetary Damages:


reliance damages
(1) This type of monetary award is designed to:
Restore the agreed party to the position he was in prior to the contract

(2) The aggrieved party’s reliance interest is measured by:


(a) any expenditure made in preparation for performance or in actually performing (out of pocket
expenses)
(b) Less:
Any loss the breaching party the agreed party would have suffered even if the contract had been fully
performed.

EXAMPLE:
A agrees to buy B’s car. On the faith of the promise, A pays B a $2,000 down payment for the car and
enters a 12 month lease agreement with C for a parking space for $2,400. The reliance interest of A
would be $4,400, the amount it would take to compensate A for what he’s lost on the faith of B’s
promise.

(3) Reliance damages may be available where:


Expectation damages are unavailable (e.g. such as when they are too uncertain or speculative)

alfrid.houston@gmail.com page 80/95


EXAMPLE:
Inventor signs a contract with Railroad Co. for shipment of a new stove Inventor plans to display at a
manufacturers’ convention, and she pays fees to the convention hotel for a room and for display
space, as well as an exhibition fee to the convention sponsor. If Railroad Co. breaches the contract by
failing to deliver the stove until after the convention, Inventor will be unable to recover damages for
opportunities she may have lost due to her inability to showcase the stove, as calculating the
probability and value of those opportunities would be too speculative. However, Inventor would be
entitled to reliance damages (i.e., a return of the Railroad Co.’s fee, plus compensation for the amounts
she paid to the hotel and convention sponsor).

d. Third type of Monetary Damages:


restitutionary damages
(1) The restitutionary damages interest is:
The value of the benefits conferred upon the other party in the transaction

EXAMPLE:
Where the buyer of a car makes a $2,000 down payment to the seller and a $2,400 payment to the
lessor of a parking space, the buyer’s restitutionary interest is limited to a recovery of the $2,000
down payment because the parking space rental payment was made to a third party. Although both
of these “out of pocket” expenses are recoverable as reliance damages, only the down payment is
available as restitution.

(2) The aggrieved party’s restitutionary interest will be measured, in the court’s discretion and as justice
requires, by either:
(a) the reasonable value or cost of the benefit conferred
(b) the extent to which the other party’s property has increased in value because of the performance
rendered

EXAMPLE:
Contractor built a Turkish bathhouse on Homeowner’s property. The market value of the services is the
cost to Homeowner of hiring another contractor to do the work, which equals $30,000. The value added to
Homeowner’s property is the net increase in the home’s value from the addition of the bathhouse, which
equals $20,000. A court calculating Contractor’s restitutionary interest could award him either amount,
considering what relevant precedents permit and what justice requires.

(3) An aggrieved party is likely to elect to recover restitutionary recovery when it would exceed the amount
recoverable based on his expectation interest, and:
That is mostly likely to arise in the context of a losing contract

EXAMPLE:
General contractor breaches its contract with Subcontractor when Subcontractor is halfway finished
with its work under the subcontract. Subcontractor is entitled to seek from the breaching General
either expectation damages or the contract price less any amounts already paid and also less any
savings due to the shortened performance. Alternately, the party could seek restitutionary damages
or the market value of the services already rendered at the time of breach. Even if Subcontractor had
a “losing contract,” it is nevertheless entitled to recover its restitutionary interest.

(4) An important limitation:


Restitution is available in the case where the aggrieved party has partially performed but not

alfrid.houston@gmail.com page 81/95


available where it has fully performed.

EXAMPLE:
Subcontractor completes contractual performance and General Contractor refuses to pay.
Subcontractor has no right to seek restitutionary damages and is entitled to recovery of the contract
price only.

e. Liquidated Damages Provisions


(1) Parties are free to include among the terms of their contract a liquidated damages clause, designed to:
Provide for damages of their own choosing in the event of a breach

(2) Such a provision is:


(a) enforceable if:
The court finds it to be a valid liquidated damages clause designed to compensate for breach
(b) unenforceable if:
The court finds it to constitute a penalty, designed to punish a breach

(3) The test for determining whether a clause in a particular contract is a valid liquidated damages provision has
three prongs:
(a) Prong #1:
Did the parties intend for the clause to operate as a liquidated damages clause or as a penalty?
1) Modern decisions downplay the importance of this prong, and emphasize the other two.

(b) Prong #2:


Was the clause reasonable at the time of contracting in relation to the anticipated harm?

1) The key question is:


Whether there was an anticipated harm that would be difficult to prove?

EXAMPLE:
Repudiation of a commercial lease by an “anchor” tenant in a shopping mall is likely to cause reductions
in pedestrian traffic to other stores as well as harm to the mall’s reputation and attractiveness as a
shopping “destination.” Because the extent and monetary value of those harms would be difficult to
establish in court, a liquidated damages clause designed to award the mall owner a sum that represents
an inexact but reasonable forecast of those harms would be appropriate.

(c) Prong #3:


Was the clause reasonable in relation to the harm and losses that actually occurred in the breach?

EXAMPLE:
If the shopping mall tenant repudiates the contract shortly after entering it, and the mall owner is able to
secure a substitute “anchor” with minimal delay, a clause guaranteeing the owner significant liquidated
damages would be unreasonable in relation to the actual harms caused by the breach.

(4) Anticipated versus Actual Harm


(a) When a particular clause satisfies the second prong of the test (anticipated harm) but not the third

alfrid.houston@gmail.com page 82/95


prong of the test (actual harm), there is a split as to whether courts will enforce the clause:
1) The majority rule:
The clause is still valid and enforceable
2) The minority rule:
Clause is unenforceable penalty

(5) If the court concludes that the liquidated damages clause is in fact a penalty:
Then the clause is stricken from the contract and damages will be recoverable in accordance with all
default rules with remedies and damages.

2. Monetary Damages under the UCC


a. Most of the rules governing remedies under the UCC reflect the same underlying principles that apply in common
cases—in particular, the right of an aggrieved party to expectation damages and the duty of an aggrieved party to
mitigate—but they differ in their details because they are specifically designed for application to the sales of
goods context.

b. Seller’s Remedies
(1) The seller’s right to recover depends on:
Whether the goods have been delivered and accepted by the buyer

(a) To the extent goods have been delivered to and accepted by the buyer:
The seller’s remedy is for the contract price
(b) To the extent goods have not been delivered to and accepted by the buyer (because buyer has
wrongfully rejected them or repudiated):

1) If the seller has re-sold:


The damages are the difference between the contract price and the resale price
2) If the seller has not re-sold:
The damages are the difference between the contract price and the market price

3) Lost Volume Sellers


a) A lost volume seller is one whose supply of goods exceeds the demand for the same. In such
case, the above differential formulas will not fully compensate for the seller’s damages.
b) The UCC accordingly permits lost volume sellers to recover:
the profit they would have made on the lost sale
c) To recover lost profits, the seller must be able to show:
i) that he could have made the sale to both the breaching buyer and the resale buyer

ii) that it would have been profitable to make those sales

iii) probably he would have made the additional sale even absent the buyer’s breach

(c) Incidental Damages


1) Whether or not the seller resells, he is also entitled to recover incidental damages, which are:

alfrid.houston@gmail.com page 83/95


The cost associated with getting stuck with the goods as well as the costs of resale

2) However, the seller’s damages will be reduced by:


Any amount reflecting expenses avoided in account of the breach

c. Buyer’s Remedies
(1) The buyer’s right to recover depends on:
Whether the buyer has covered (purchased replacement goods)

(a) If the buyer covers:


Damages are the difference between the contract price and the cover price

(b) If the buyer does not cover:


Damages are the difference between the contract price and the market price

(c) Additionally, the buyer is free to seek incidental and consequential damages.
1) Incidental damages:
The cost associated with securing cover (eg transportation etc)

2) Consequential damages: - foreseeability


The peculiar cost arising to the buyer because of a particular need or use for the goods in Q, such
as lost profits - foreseeability applies.

3) The buyer’s damages will be reduced by:


Any expenses avoided because of the breach

(2) Difference in Value Damages for Accepted Goods


(a) This measure is available if the buyer receives and accepts nonconforming goods from the seller.
(b) Under this measure, the buyer is entitled to recover the difference between:
1) the value of the goods contracted for
2) the value of the goods received

3. Equitable Remedies
a. First type of Equitable Remedy:
specific performance
(1) This is an extraordinary remedy by which a breaching party is ordered to perform.
(2) This remedy is only available to an aggrieved party when:
A monitory award is considered inadequate
(3) Where Specific Performance Is Generally Available
(a) Money damages are generally presumed to be inadequate when a party is purchasing either:
1) unique objects
a) For example:
works of art and precious things

alfrid.houston@gmail.com page 84/95


2) real property
3)
(b) Equity Considerations
1) We are dealing with an equitable remedy and so the question is committed to the judge’s discretion. In
so deciding, courts are likely to consider the following factors:
a) whether the aggrieved party has “clean hands;”
b) whether the terms of the contract are fair;
c) whether the terms of the contract in question are sufficiently definite;
d) whether performance by the aggrieved party can be reasonably assured; and
e) whether specific performance would be in the public interest.

(4) Where Specific Performance is not Available


(a) contracts for personal services
(b) contracts requiring ongoing co operation between the parties, e.g. construction contracts, landlord-
tenant contracts

(5) Specific Performance under the UCC


(a) The UCC liberalizes the rules governing the availability of specific performance for sales of goods in
two ways:
1) Liberalizing the “uniqueness” requirement:
Under the UCC, specific performance is available to a B who has adequately searched but unable
to find reasonable substitutes so unable to cover the breach by the seller

2) Liberalizing the “capable of immediate performance” requirement:


Under the UCC, specific performance is allowed in output contracts ands requirement contracts
which are classic long-term relationships which require co-operation between the parties.

b. The second type of Equitable Remedy:


negative injunction
(1) Negative injunctions are:
Orders by the court prohibiting the breaching party from taking a particular action

(2) The most common and important area for negative injunctions is:
The employment setting
(3) The availability depends on whether the former employer is seeking mid-term or post-employment relief.
(a) Mid-Term Relief
1) When an employee is under contract for a specified period of time and the employee breaches the
contract by departing before the end of that period, a negative injunction will be available to prevent
the employee from competing if:
The employee services are unique or extraordinary

EXAMPLE:
In the famous case of Lumley v. Wagner, an opera singer under contract to sing at Her Majesty’s
Theatre for a three-month period was persuaded to depart mid-contract to begin a concert series at a
competing venue. Because her services were unique and extraordinary, the court granted a negative
injunction barring her from performing at any competing venue for the duration of the contract term.
alfrid.houston@gmail.com page 85/95
2) Although the presence of a specific contractual provision establishing exclusive employment during
the period of time in question will aid the employer is securing a negative injunction:
Most courts will imply such a term for the period of employment in the contract

(b) Post-Employment Relief-more likely to be tested. Comes up a lot


1) Enforcement of employment contract provisions that prohibit post-employment competition
against the employer have become a common source of negative injunctions.
2) The validity of such non-compete clauses will depend on three factors or considerations:
a) Consideration #1:
there are significant business justifications for enforcing post-employment restraints

EXAMPLE:
An employee with access to trade secrets and entrepreneurial knowledge.
b) Consideration #2:
is the scope of the non-compete clause reasonable in both duration and geographical reach?

c) Consideration #3:
is there an express provision? – courts will not employ a non compete, unless you have the
extraordinary skill argument above.
B. Other Possible Remedies
1. Promissory Estoppel
a. The type of interest a party may recover under a claim of promissory estoppel depends on the jurisdiction, as
some courts award:
(1) expectation damages
(2) reliance damages
(3) choose on a case by case basis and tailor the remedy to the injustice at issue

2. Restitution and Unjust Enrichment-frequently tested


a. Restitution is that alternative to expectation damages that a party can recover for a breach of contract.
b. But there are contexts where restitution might be available to a party even when the other party has not breached
a contract:
(1) Benefits Conferred under a Failed Contract
(a) When a party bestows benefits on his trading partner in connection with what turns our to be a “failed”
contract (e.g., incapacity, fraud, duress):
The party bestowing the benefits may recover their value via restitution

(2) Benefits Conferred by a Breaching Party


(a) a party that breaches the contract may recover the benefits it conferred on the non-breaching party so
long as there is an off set for any damages caused by the breach.

EXAMPLE:
A works for B under a 12-month contract and, without advance notice, breaches the contract after 11
months in order to other accept other employment. A is entitled to restitution of the reasonable value of
the services rendered, subject to offset for B’s damages—that is, the cost of hiring a replacement for A
at the last minute.

alfrid.houston@gmail.com page 86/95


(3) Emergency Benefits Conferred by a Health Care Professional
(a) The general rule is that a person who bestows benefits without request from the benefitting party:
Is considered an officious intermeddler not entitled to any recover since there was no opportunity to
decline

(b) However, an exception applies to:


Doctors and other health care professionals who provide emergy health care to a patient who is unable
to consent

(4) Benefits Conferred by Mistake-tested


(a) A person who mistakenly confers benefits to another party may be entitled to restitution.
(b) In this situation, the following will be considered:
1) the blameworthiness of the error
2) whether the recipient was aware of the error in time to prevent it
3) where the recipient availed himself of the benefits at issue

EXAMPLE:
After a hurricane, Owner hires Contractor to perform repairs on his storm-damaged home. Because
of confusion caused by damage to street signs and mailboxes, Contractor performs the repairs on
the wrong house. Contractor is entitled to restitution from the benefiting homeowner if the latter was
aware of Contractor’s mistaken efforts and remained silent.

3. Agreed-To Remedies
a. Parties may also contract out of the legal and equitable remedies available under the law, by specifying agreed-to
remedies in the contract.
b. These remedies typically take two forms:
(1) liquidated damages provisions
(2) provisions limiting or excluding damages

c. Provisions Limiting or Excluding Damages


(1) There are different types of exclusive remedies provisions, including:
(a) provisions that limit or alter the measure of damages available

EXAMPLE:
An exclusion of consequential damages.
(b) exclusive remedies – limits to repair or replacement of defective goods

(2) Such provisions are generally enforceable unless they:


Are unconscionable or fail of their essential purpose
(3) Limitation of consequential damages for personal injury:
In the case of consumer goods, this is prima facie unconscionable

alfrid.houston@gmail.com page 87/95


APPLICATION

Question 7
Marge Billings, owner of a gas station, entered into a written contract with Hudson Construction to build a store on her lot.
Under the terms of the contract, Billings was to supply detailed plans and specifications for the store, and Hudson would
build the store for $300,000. According to a provision in the contract, all changes to the contract had to be in writing.
Hudson Construction broke ground and worked quickly on the store. About half way through the project, Steve Hudson, owner
of Hudson Construction, realized that construction would cost more than he originally estimated as a result of delays in
receiving materials ordered and extreme shortages in construction materials due to a recent massive hurricane in the area.
Hudson explained the situation to Billings. He indicated he could not afford to take a big loss on the construction of the store and
estimated he could not complete the store for less than $450,000. Billings refused to pay more, and Hudson ceased
construction.
If sued for breach of contract, Hudson’s best defense is
A) duress, coercion, and impracticability.
B) impossibility of performance.
C) substantial performance.
D) commercial impracticability.

APPLICATION

Question 8
A clause in a locker rental contract at a local health club reads, “Renters canceling their locker contract after using their locker
shall pay liquidated damages in the amount of $5.00 per day for the remainder or unexpired portion of the term of the locker
rental agreement.” Lockers rent for $50.00 per month or $500.00 per year. A renter with a year-long contract leaves 29 days
early and the health club demands payment.
If the club sues, the most likely outcome is that
A) because the renter willfully breached the contract, liquidated damages will be allowed.
B) because the renter has substantially performed on the contract, the club will not be entitled to damages.
C) because the contract remedy is disproportionate to the injury, it is unlikely the court will enforce the clause.
D) because the parties clearly agreed to a specific measure of damages, the court will accept that measure.

RECAP QUESTIONS:

Please take a few minutes and answer the following questions. Try to answer these questions without looking at your Bar
Notes. If you must go back to your notes, try rewriting the answer in your own words. This exercise will help improve your
memory retention of these concepts.

1. What are expectation damages?

2. Describe four situations when the right to recover full expectation damages may not be available.

3. When is specific performance NOT available?

alfrid.houston@gmail.com page 88/95


X. THIRD-PARTY BENEFICIARIES- REGULARY TESTED

A. In General
1. In a typical contract, the parties promise performances to each other. If one party refuses to perform, the other has
standing to bring a claim for breach because the parties are in contractual privity. In some contracts, however, one of
the parties promises a performance that will benefit a third party (a third party beneficiary).
2. The critical issue in third-party beneficiary law:
Is the circumstances in which the third-party beneficiary has standing to enforce the contract

B. Classification of the Third-Party Beneficiary


1. The right of an aggrieved third-party beneficiary to bring an action against a breaching promisor or promisee will
depend upon the classification of the beneficiary, which differs under the First and Second Restatements (although
the practical effect is largely the same).

2. First Restatement
a. The first Restatement placed third-party beneficiaries into one of three categories:
(1) Creditor beneficiary:
A promisee seeks a performance from the promisor that will satisfy an obligation owed to a third
party

(2) Donee beneficiary:


A promisee seeks a performance from the promisor in order to make a gift of that performance to
a third party

(3) Incidental beneficiary:


Third parties who will benefit from a promisor’s performance as a practical matter but are neither
creditor or donee beneficiaries.

3. Second Restatement-has replaced the First


a. The Second Restatement eliminates the “creditor” and “donee” beneficiary terminology, and instead puts third-
party beneficiaries into one of two possible categories:
(1) intended beneficiaries
(2) incidental beneficiaries
(a) The key factor in determining whether a party is an intended beneficiary is:
“The intent to benefit” test
(b) Incidental beneficiaries, similarly to the First Restatement, are:
alfrid.houston@gmail.com page 89/95
Third parties who will benefit from the promisor’s performance as a practical matter but are not
intended beneficiaries.

EXAMPLE:
The residents who live in the neighborhood where Lawyer’s school is located will be benefited by the
mural, but none of them can sue if Artist does not deliver because they are incidental beneficiaries.
Only the school has a claim.

C. Rights of Parties to Enforce the Contract


1. Third-Party Beneficiary’s Rights
a. Under both the First and Second Restatement, an incidental beneficiary:
does not enjoy any right to seek enforcement of the contract

b. Third-Party’s Rights against the Promisor


(1) Any third-party beneficiary, other than an incidental beneficiary, has a right:
To secure enforcement of the agreement from a breaching promisor, i.e. can sue the promisor

c. Third-Party’s Rights against the Promisee


(1) A third-party beneficiary has no rights against the promisee in connection with the promised performance
under the new contract.
(2) Instead, a third-party beneficiary will only have rights against the promisee based on:
Whether there is a prior obligation between the promisee and the third-party beneficiary

d. Vesting of the Right to Sue


(1) The parties to a contract are free to modify or rescind it by mutual consent, and they may modify or rescind a
third-party beneficiary provision without the beneficiary’s consent unless and until the beneficiary’s rights
under the contract have vested.
(2) Vesting occurs with regard to an intended beneficiary when:
(a) Situation #1:
When the beneficiary brings suit on the matter

(b) Situation #2:


The beneficiary changes its position in justifiable reliance on the contract

EXAMPLE:
Landlord has had it with Artist and is ready to start eviction proceedings. The landlord then learns that
Artist has arranged that Lawyer’s payment for the mural be paid the landlord . In light of this, the
landlord decides not to continue with the conviction proceedings. At this point, because the
landlord/beneficiary has changed his position in justifiable reliance, the landlord/beneficiary’s rights
have vested.

(c) Situation #3:


The beneficiary manifests assent to the contract at the request of the promissee or promisor.

EXAMPLE:
Patron (the lawyer) speaks to representatives of her alma mater, informing them about the mural that
she has arranged to be painted for the school. The representatives gladly assent to the painting. Once
alfrid.houston@gmail.com page 90/95
the school says it will accept the mural, it is too late to modify the contract with the artist.

(d) Situation #4:


If the rights of the beneficiaries have vested under the express terms of the contract

EXAMPLE:
Under the terms of her life insurance policy with Hartford Insurance, A’s designations of beneficiaries
become irrevocable upon A’s demise or adjudication of incompetence. The rights of C, a designated
beneficiary, against Hartford Insurance will vest upon the occurrence of either event.

e. Defenses Available to Promisor


(1) Because a third-party beneficiary’s rights are entirely dependent on the underlying contract:
Any valid defenses the promisor has against the promisee, they are also effective against the third
party beneficiary

EXAMPLE:
If Lawyer and Artist enter into a contract in which Lawyer agrees to pay artist for the painting of a
wall mural, and Artist in fact does not paint that mural, there is a failure of consideration and
Lawyer’s obligation to pay is discharged. Moreover, Lawyer can raise that as a defense against
Artist’s landlord, if that landlord sues as a third-party beneficiary of the contract.

(2) The promisor may not assert defenses:


Based on separate transactions with the promisee.

EXAMPLE:
Before Lawyer entered into a contract with Artist for the painting of the mural, Lawyer had loaned
Artist money. Artist has not repaid the loan as promised. In an action between Artist and Lawyer,
Lawyer has a right of setoff. But if Artist’s landlord sued as a third-party beneficiary to the contract
for the mural, Lawyer could not raise the setoff as a defense.

2. Promisee’s Rights against the Promisor


a. When the promisor does not perform, the promisee has a claim for breach of contract against the promisor.
b. But if the promisor’s performance is intended to benefit a donee beneficiary:
then the promisee has not suffered any economic loss and cannot recover damages. May obtain specific
performance if appropriate (but here-personal service)

EXAMPLE:
Law school is the donee beneficiary of a contract between Lawyer and Artist, where Artist is the promisor (promising a
performance—the painting of a mural) and Lawyer is promisee. If Artist refuses to perform, the school can sue Artist.
However, a suit by Lawyer is problematic. As to what Lawyer’s remedy might be, Lawyer’s loss is not monetary and so
expectation damages will not be available. Further, since the underlying contract is one for personal services, specific
performance would not be available either.
c. If the promisor’s performance is intended to benefit a creditor beneficiary:
the promisee may secure specific performance of the promisor’s obligation.

(1) A claim for money damages would, however, expose the promisor to the possibility of double liability, since
the promisor is also liable to the third-party beneficiary.
alfrid.houston@gmail.com page 91/95
(2) Accordingly, some courts refuse to allow the promisee to recover damag es against the promisor:
Unless the promisee has already made payment to the beneficiary to cover the default

EXAMPLE:
Artist owes money to the landlord. Artist and Lawyer enter into a contract whereby Lawyer agrees to
pay artist’s landlord, who is then the creditor beneficiary of that contract. In this instance, the
Lawyer is the promisor (promising performance—the paying of Artist’s rent to the landlord). If
Lawyer refuses to pay, Artist could sue Lawyer for specific performance of Lawyer’s promise to pay
the creditor beneficiary. The court may require Artist to first pay Landlord before proceeding,
however.

XI. ASSIGNMENT OF RIGHTS AND DELEGATION OF DUTIES

A. Assignment of Rights
1. An assignment is:
A transfer of a right to receive a performance under a contract
2. To make an effective assignment of a contract right, the owner of that right must:
a. manifest an intention
b. make a present transfer of the existing right

3. Rights to Be Assigned
a. The general rule is that:
all rights are assignable, subject to:
(1) Exception #1:
If the assessment would materially alter the risks to or obligations of the other party

EXAMPLE:
A and B are parties to a requirements contract under the terms of which A is obligated to supply B
with B’s monthly requirements for widgets. B’s rights under the contract are not assignable to C if
C’s monthly requirements would greatly exceed B’s.

alfrid.houston@gmail.com page 92/95


(2) Exception #2:
When the obligor has a personal interest in rendering the performance in question to the oblige
and not a third party

EXAMPLE:
A student attempts to assign his right to a private tutor who is picky about his students. Because the
tutor has a personal interest in who he tutors, the contract is not assignable.

(3) Exception #3:


Violate applicable law or public policy

EXAMPLE:
A state statute prohibiting assignment of wages.

(4) Exception #4:


The assignment is prohibited by a contract
(a) However, most courts will treat an assignment in violation of contractual restriction as:
A breach of contract by the assignor but not a basis for nullifying obligee’s rights (i.e. it takes away the
right but not the power to assign).

EXAMPLE:
If a bar course has a “no assignment of bar course rights” provision in its contract and one of its students
assigns the contract right to another student, the bar course has an action against the student who
breached the contract but must provide the bar course to the assignee.

4. Assignment for Value Contrasted with Gratuitous Assignment


a. Assignment for value:
valid against the obligor and cannot be revoked by the assignor

b. A gratuitous assignment has the following legal effects:


(1) Between the assignee and the obligor:
Its valid against the obligor who cannot claim lack of consideration as a defense

(2) Between the assignor and the assignee:


Under the law of gifts, an executory gift is revocable but an executed gift is not (a gift is executed
when there is an intention coupled with actual or symbolic delivery).

EXAMPLE:
A recent graduate of law school has paid for a bar course but decides instead to join the clergy. As a
gesture of love, he transfers the right to the bar course to a friend not for value but as a gift.
5. Rights and Obligations of the Parties after Assignment

a. Rights of Assignee against the Obligor


(1) The basic rule is that:
(a) an assignee gets whatever right to the contract his assignor had

alfrid.houston@gmail.com page 93/95


EXAMPLE:
If assignor paid for the full course, that is what the assignee is entitled to.
(b) assignee takes subject to whatever defenses the obligor could have raised against the assignor.

EXAMPLE:
If assignor has not paid for the full course, the bar course can raise that as a defense against the
assignee.

(2) Payment to Assignor


(a) The obligor’s payment to the assignor:

b. Rights of Assignee against Assignor


(1) Unless a contrary intention is manifested, one who assigns or purports to make an assignment for value
impliedly warrants two things to the assignee.
(a) he will do nothing to defeat or impair the value of the assignment and has no knowledge of facts that
will do so

(b) the right as assigned actually exists and is subject to no limitations or defenses good as against the
assignor

EXAMPLE:
If the bar course is assigning accounts receivable to some creditor party knowing full well that it has
failed on its performance, it will violate this implied warranty.

NOTE: In and of itself, an assignment does not operate as a warranty to the assignee that the obligor is solvent
or that the obligor will perform his obligation.

B. Delegation of Duties

1. A delegation occurs when:


A third party to agrees to satisfy a performance obligation owned by one of the parties to the contract

alfrid.houston@gmail.com page 94/95


2. Rights of the Obligee against the Delegator
a. While an assignment of rights effectuates a transfer of those rights to a third party (upon assignment, the assignee and
not the assignor has rights under the original contract), a delegation of duties does not operate as a “transfer” of those
duties from the delegator to the delegatee.
(1) absent novation, a delegation does not relieve the delegator from his obligations under the contract, i.e. the
obligee can still sue the delegator.

3. Liability of the Delegatee


a. To the delegator (the Professor, in our example):
if the delegation was for consideration, the delegator has a breach of contract action against the delegatee who
doesn’t perform. If without consideration – no.

b. To the obligee (the bar course, in our example):


if the delegation was for consideration, the obligee can bring an action against the delegatee as an intended third
party beneficiary of the delegation.

4. Delegable Duties
a. The general rule:
all contractual duties are delegable
b. The exceptions:
(1) Exception #1:
When the performance in question is personal

EXAMPLE:
Nanny is under contract to provide child care services to B’s family. Her performance obligation
cannot be delegated to a third party.
(2) Exception #2:
When the contract prohibits delegation.

APPLICATION

Question 9
At the beginning of March, George entered into an agreement with Pat, a local housepainter, whereby Pat would paint
George’s house by the end of December. The contract specified that Pat would use paint from Tom’s Hardware, as Tom’s
sells the highest quality paint available in the area.
After the contract was signed, Pat told Tom that he had another contract coming. Tom thanked Pat and ordered paint in
expectation of Pat’s needs. By November, a revolutionary new house paint came to market but was not available at Tom’s
Hardware. Pat and George agreed that Pat should use this new paint. Tom is upset at the loss of the sale, and wishes to
enforce his rights under the contract.
Will Tom be successful?-here he is incidental beneficiary.
A) Yes, because he was a third-party beneficiary.
B) Yes, because he ordered paint in anticipation of Pat’s needs.
C) Yes, because Tom assented to the benefit of the contract.
D) No, because the reason for the contract was to paint George’s house.

alfrid.houston@gmail.com page 95/95


APPLICATION

Question 10
Josh, the owner of Josh’s Car Wash, was tired of customers assigning their right to a specialty car wash and detailing
package to others. To avoid this problem in the future, Josh had his contracts rewritten to specify the name of the customer
and to include the sentence, “Contract shall not be assigned.” One such contract called for Josh to wash and detail Al’s four-
door sedan for $200. Al had an emergency at work the day he was to drop off his car at Josh’s Car Wash. On his way out the
door, Al executed a document assigning his rights under the contract to his neighbor, Bill. Bill paid Al $250 for the
assignment. Bill arrived at Josh’s Car Wash with the signed assignment and requested that his two-door sport coupe be
detailed and washed. Josh refused to honor the assignment.
Has Josh breached the contract?
A) No, because the original contact said assignments are prohibited.
B) No, because the name of the party is a material term.
C) Yes, because the assignment was in writing supported by consideration.
D) Yes, because a sports coupe is comparable to if not smaller than a four-door sedan.- assignment doesn’t materially
change the obligations.

RECAP QUESTIONS:

Please take a few minutes and answer the following questions. Try to answer these questions without looking at your Bar
Notes. If you must go back to your notes, try rewriting the answer in your own words. This exercise will help improve your
memory retention of these concepts.

1. Who are the parties in a Third Party Beneficiary scenario?

2. What must the owner of a contract right do to properly assign that right?

3. In what instances are contracts rights not assignable?

4. After a delegation of duties, what obligations , if any, remain with the delegator?

alfrid.houston@gmail.com page 96/95

You might also like